sábado, 19 de abril de 2014

É correto afirmar que matéria é energia vibrando numa frequência baixa?

Não. Matéria não é energia, de modo nenhum. Energia é um atributo, uma propriedade, Matéria é um tipo de entidade substancial do Universo. Energia não é. Nada é "feito" de energia, mas "possui" energia (ou não). Não existe "energia pura". O que existe é a energia que algo possua. A equação E-mc² não mostra a equivalência entre matéria e energia, mas entre massa e energia, pois massa também é um atributo, uma propriedade. Matéria é um conglomerado de quantizações fermiônicas de campo, isto é, de partículas que possuem spin semi-inteiro. Uma quantização de campo ocorre quando o campo quantizado "ondula", com certa frequência, havendo uma superposição de ondas que restringe a existência da onda a uma região restrita do espaço, chamada "pacote de onda", que é a partícula em questão, como os quarks e os léptons. Mas essa frequência não é nada baixa. E o que vibra não é a energia, mas o campo. Claro que essa vibração possui energia, mas não "é" energia.

Professor, cursar cinema vale a pena? É algo que eu amo muito, mas que ninguém aprova.‎

"Tudo vale a pena quando a alma não é pequena". Claro que vale fazer cinema. Se é o que você gosta é o que deve fazer. É só não se preocupar com o dinheiro que vai ganhar. Pode ser que precise arranjar alguma fonte de renda provisória até se firmar como diretor, produtor, roteirista ou ator. Mas vá em frente e realizes o seu sonho. É o que digo para todo mundo que quer fazer seja o que for por que tenha grande fissura em fazer. O que nunca recomento é que vá dedicar a vida a fazer algo por que não se sinta atraído só porque seja o que dá dinheiro.

o que faz a particula boson de higgs ativa, existir,o que tem alem dela, de onde ela vem

Todas as partículas existentes são quantizações de campo, que preenche o espaço, constituindo o vácuo, que não é vazio. Esse campo foi o conteúdo que surgiu no surgimento do Universo e que gerou o espaço que ocupa, que logo passou a se expandir e cuja expansão gerou o tempo. Não há nada mais primitivo do que esse campo. Ele não é feito de nada além. Ele é o componente básico de tudo o que é feito. Mesmo a teoria das cordas considera que as cordas são quantizações de campo. Há o campo da matéria e os campos das interações, que, a princípio, eram indistinguíveis e com a expansão e esfriamento se separaram por quebras de simetria. Isso logo nas primeiras frações de segundo de existência do Universo. O bóson de Higgs é uma quantização do campo de Higgs que é o campo de interação que produz a inércia das partículas materiais e, daí, sua massa. Quantização de um campo é uma concentração do dito campo em uma região restrita, caracterizada por valores definidos de certas propriedades, como carga, spin e outras, que só existe com esses valores.

Como aprender a se expressar melhor?‎

Lendo muito, estudando gramática (isso mesmo!) e, principalmente, expressando-se sem medo de errar e corrigindo os erros sempre e sempre. Quem não arrisca não petisca. Quando ler, ler com atenção, examinando a construção dos argumentos e o vocabulário usado. Nunca passar por cima de uma palavra cujo significado não se conheça sem buscá-lo. Assim é que se progride. Não se pode ter o mínimo resquício de preguiça mental.

Professor, quero muito fazer física, mas tenho medo do curso. Vale a pena arriscar ? O que o senhor acha ? E, aliás, não estou acostumado com exatas. Obrigado professor!‎

Mas a Física não é um bicho de sete cabeças (só seis). De fato, há muita grande necessidade de Matemática no estudo da Física. Mas Física é muito mais do que Matemática. Matemática é uma ferramenta para a Física. O mais interessante, contudo, não é a Matemática, mesmo que ela seja imprescindível para a Física. É a compreensão do funcionamento da natureza em seus aspectos mais profundos. Essa é a grande beleza da Física. Como sou uma pessoa totalmente deslumbrada pela Física (como também por outros assuntos), acho que, de tudo o que se possa dedicar a se fazer, entender como a natureza funciona é o mais fantástico. E não é pena nenhuma, pelo contrário, é uma delícia. Por isso vale, sim.

Como explica as as semelhanças entre as pessoas e seus respectivos signos uma vez que não acredita em Astrologia?‎

Essa semelhança não existe. Só acontece, por coincidência em alguns casos e, mesmo assim, em alguns aspectos. Além do mais, não há nada que seja capaz de mostrar uma relação causal entre a data do nascimento e a personalidade de alguém. Há pessoas das mais variadas personalidades e outras características que nascem no mesmo signo. Outra coisa é que as previsões astrológicas de horóscopos, se feito um levantamento estatístico sério, mostram-se completamente falsas. Astrologia é um grandíssimo engodo.

Por que foi existir vida logo na Terra e não em outro planeta do nosso sistema? E os primeiros seres vivos surgiram do nada?‎

Por coincidência, ora. Se estamos aqui é porque aqui que surgiu a vida. Se tivesse surgido em outro lugar, é lá que estaríamos. Pode ser que tenha surgido, também em outros lugares. Quanto aos planetas do Sol, só a Terra reúne as condições propícias para a vida. Pode ser que Marte, outrora, tenha tido, também, essas condições. Ou alguma lua de Júpiter e Saturno. O resto é inviável. Os primeiros seres vivos não surgiram "do nada". Nada surge do nada, mas pode surgir de nada. Mas também não surgiram de nada. Surgiram da matéria que havia na Terra, que, por acaso, se organizou de tal forma que deu início à replicação por conta própria, bem como o autoprovimento de energia, que é a vida.

o que você me diz da geofísica? é um bom ramo no mercado de trabalho? e quanto ao conhecimento individual, acrescenta suficientemente bem?‎

Não conheço o mercado de trabalho do geofísico, em termos de remuneração. Mas sei que é importante na prospecção de petróleo e de jazidas minerais outras. Bem como no estudo da previsão de sismos. Logo é de grande utilidade. Quanto a dar o prazer intelectual da fruição do conhecimento, isso depende muito do pendor de cada um. Há quem adore geofísica e quem deteste. Trata-se de uma interface entre a física e a geologia. Portanto envolve o conhecimento de ambas, o que é mais abrangente do que apenas ser geólogo ou ser físico.

Professor, lhe fiz essa pergunta há uns 10 dias, mas você não retornou: poderia explicar ou explanar sobre a detecção dos "tetraquarks"? http://www.inovacaotecnologica.com.br/noticias/noticia.php?artigo=lhc-confirma-existencia-novo-tipo-materia&id=010130140414#.U06T0FcvmSN‎

A cromodinâmica quântica (teoria dos quarks) prevê a possibilidade de tetraquarks e até partículas com mais quarks. O que foi descoberto é uma dessas. Mas a teoria prevê propriedades, como a meia vida, que a experiência mostrou estarem incorretas. O que se precisa, pois, é refinar a teoria para que ela se adeque aos fatos experimentais. É assim que a ciência progride. Nenhuma teoria científica é definitiva. Não se tem certeza de nada que se sabe. Isso é que é a grande beleza da ciência. Corrigir-se a si mesma à medida que novos conhecimentos vão sendo obtidos.

Como ser criativo?

Tentando, inventando, ousando. Sem se preocupar em acertar, sem medo de errar, sem ligar para as decepções e as críticas, sem esmorecer com os fracassos. É como aprender a andar de bicicleta ou a nadar. Se não experimentar fazer, nunca se aprende. Não há dicas específicas.

Filosofia ou Física, qual fazer? Por quê? Boa noite professor!‎

Boa noite. Faça Física formalmente e Filosofia informalmente. Depois, se der, faça Filosofia formalmente também. A Física é a mais filosófica das ciências e a Filosofia, sem conhecimentos físicos, matemáticos, biológicos, psicológicos, sociológicos, políticos e econômicos fica completamente coxa. Um filósofo tem que ser uma pessoa que domine amplamente todos os conhecimentos humanos. Isso faz parte intrínseca da Filosofia. Só que, desses todos, os que requerem uma dedicação formal para serem dominados são mais a Física e a Matemática. Mas quem cursa Física, necessariamente, vai estudar muita matemática. Por isso é melhor fazer um Curso de Física e se dedicar à Filosofia do que o inverso.

O que você acha do Jair Bolsonaro?‎

Um reacionário, retrógrado, preconceituoso, conservador, mesquinho, autoritário. E o pior: acha que isso tudo são grandes virtudes e não defeitos.

O que você acha do Homo Ludens? O conceito de jogo poderia ser empregado para estimular a educação, por exemplo? É possível jogar sem competir? A competição é sempre algo ruim?‎

Jogos são algo interessante e proveitoso. Competição não. Jogos não precisam ser competitivos. Não se precisa jogar para derrotar ninguém. Deve-se jogar para vencer apenas as próprias limitações. Isso é bom. De fato, o ser humano, mesmo adulto, tudo o que faz o faz como uma brincadeira, por mais sério que pareça ser. Até a guerra, a política e os negócios são jogos. Mas não precisam ser jogos de competição. Podem ser jogos de superação. E podem envolver colaboração entre os jogadores. Para que todos vençam. O bom é isso. Não é bom que ninguém seja derrotado. Mas não é bom que alguém consiga vencer usando meios desonestos. Esse não pode vencer. Isso não significa que outro vá derrotá-lo, mas que ele não pode lograr sucesso. A sociedade tem que impedir a vitória dos tratantes. Tampouco não é bom que se obtenha vitória sem a superação das deficiências. Não é válido que se iguale todos, pois, há diferenças. O que não pode é haver desigualdade de oportunidades. Uma vez que todos tenham as mesmas oportunidades e que ninguém vá prejudicar a ninguém, quem tiver capacidade e trabalhar com afinco vai aproveitar as oportunidades e será vitorioso, sem precisar derrotar ninguém.

Uma pessoa que acredita no Big Bang é necessariamente agnóstica/ateia? Ou acreditar no mesmo vem do agnoticismo/ateísmo em si?‎

Não. Pode considerar que o surgimento do Universo, sua expansão, sua evolução, o surgimento de estrelas, galáxias, planetas, da vida e da inteligência na Terra e, possivelmente em outros lugares tenha sido algo provocado por uma divindade, mesmo tendo ocorrido como a ciência descreve que o foi. Note que aceitar a descrição científica não é uma crença, pois ela é fundamentada em comprovações. Uma pessoa pode crer que exista alguma ou algumas divindades sem considerar que a descrição que as religiões fazem da origem do Universo, da vida e da vida inteligente sejam conforme o que as doutrinas religiosas propõem que seja.

Qual a diferença entre Astrônomo, Astrofísico e Cosmologista?‎

A Astronomia estuda a posição e movimento e Astrofísica estuda a evolução e funcionamento dos astros, seus grupamentos e o conteúdo do espaço entre eles. Cosmologia estuda a origem, estrutura e evolução do Universo como um todo.

Qual sua opinião sobre o educador Antonio Gramsci? Vejo tantas pessoas usando o termo "gramscista" de forma pejorativa, mas fui ler sobre e concordei com várias ideias defendidas por ele.

Essas pessoas, ou não sabem o que falam ou são reacionárias e favoráveis à manutenção do "status quo" de dominação das elites plutocratas sobre a população em geral. Gramsci é um pensador extremamente lúcido e coerente. Sua crítica ao marxismo provém de um idealista que queria ver o comunismo triunfante no mundo. Sua insistência na educação das massas trabalhadoras, especialmente na formação de intelectuais de origem popular é uma de suas maiores contribuições, muito bem assimilada por nosso grande pedagogo Paulo Freire. Vale a pena inteirar-se de suas concepções, mesmo que se as assimile com visão crítica. Mas ele retira os antolhos, não só dos capitalistas neo-liberais, mas também de muito comunista ortodoxo.

quarta-feira, 16 de abril de 2014

Professor, a razão pode levar ao conhecimento sem necessitar dos sentidos, mas os sentidos não podem levar ao conhecimento sem o apoio da razão?‎

Não é bem assim. Primordialmente, desde a vida intrauterina, tudo o que a mente (que é uma ocorrência do cérebro e seus anexos) adquire de conhecimento do mundo é captado pelos sentidos. Por sua arquitetura de funcionamento, o cérebro é capaz de raciocinar sobre as percepções e tirar conclusões, que são conferidas experimentalmente pelo corpo e pelos sentidos e assim vai se construindo o arcabouço de conhecimentos, tanto conceituais quanto procedurais. Desse modo é possível que o raciocínio, tanto consciente quanto inconsciente, possa construir novos conhecimentos sem o aporte direto dos sentidos nessa construção, mas com base no que já está registrado e o foi, anteriormente, por percepções advindas de sensações ou de raciocínios passados. De qualquer modo, um conhecimento não é, apenas, um registro sensorial, mas uma interpretação deles por cotejo com todo o cabedal de registros já consolidados, o que constitui as percepções, bem como os desdobramentos racionais a partir delas, isto é, a formação de juízos, abstrações, associações, e raciocínios mais complexos. Como também os sentimentos, que combinam percepções com emoções. A consciência e a auto-consciência são um tipo de percepção, a saber, a que o cérebro tem de seu próprio funcionamento e do organismo como um todo, que dá a sentimento do "eu". Mas a maior parte das operações racionais do cérebro é feita de modo inconsciente, ensejando a emergência à consciência na ocorrência denominada "intuição".

professor, há alguma diferença entre buscar o conhecimento e buscar uma verdade? se sim, poderias me explicar?

Conhecimento é uma explicação sobre o modo e a razão de algo ser e acontecer como é e acontece. Claro que se busca sempre o conhecimento verdadeiro, isto é, a explicação correta sobre o comportamento da realidade. Verdade, contudo, pode não se referir a um conhecimento, nessa acepção. Pode se referir à existência de algo, mesmo não abrangendo o seu entendimento e compreensão. Ou a algum juízo afirmativo ou negativo atribuindo algum predicado a algum sujeito, por meio de um verbo de ligação (ser ou estar, por exemplo), no caso dos juízos serem sintéticos, isto é, quando o predicado já não faça parte do sujeito, Tais juízos precisam ser veritados faticamente e nisso consiste o maior trabalho de busca da verdade. Mas não se trata de um conhecimento, epistemologicamente falando.

Professor acordei pra ver o eclipse de madrugada e o ceu estava vermelho. É por causa do eclipse ou coisa relacionada a ele, ou é normal ficar dessa cor ? Isso foi as 4h‎

Nas noites nubladas o céu reflete a luminosidade da cidade e, se tem muitas lâmpadas de sódio nas ruas, fica meio alaranjado. Se a lua estava por trás das nuvens, no período em que ficou avermelhada, aquela região deve ter ficado avermelhada também. A vermelhidão não foi visível enquanto o eclipse não ficou total porque a excessiva luminosidade da parte em que ainda batia Sol fazia um grande contraste com a parte escurecida. Quando ficou tudo na sombra completa (umbra) então se viu a vermelhidão, se não havia nuvens atrapalhando a visão.

Porque na maioria dos eclipse ou eclipses , o céu esta totalmente encoberto com nuvens ??

Não me consta que assim o seja. Mas não fiz nenhum levantamento. Isso depende do lugar em que se está.

Por que os professores não despertam o desejo ou a fascinação no aluno sobre a matéria ou a disciplina que é ensinada?‎

Porque não são fascinados por ela. Esse é o grande mal. Um verdadeiro professor tem que estar imbuído de um total deslumbramento por sua área de conhecimento. Tem que mergulhar de cabeça e querer saber tudo o que se sabe a respeito e não apenas o que tem que ensinar. Tem que fruir o máximo prazer em estudar a sua matéria. Tem que adorar, justamente, resolver os problemas mais complicados e dar as explicações para as questões mais difíceis e embaraçosas. Tem que ser um professor porque é isso que ele quer ser na vida, mesmo tendo capacidade para ser qualquer outra coisa que queira ser. E tem que ser totalmente desprendido do retorno financeiro de sua atividade. Além disso, tem que ser movido por um desejo profundo de que o conhecimento e a aquisição de habilidades em sua matéria seja algo a ser comunicado e difundido para todo mundo, para que todos também enxerguem a maravilha que é. E dominar os recursos de argumentação e comunicação capazes de propiciar tal feito.

Se identifica com os pensamentos e obra de algum filósofo mundialmente conhecido?‎

Totalmente, com nenhum. Parcialmente com David Hume e Bertrand Russell. Não me alinho com nenhuma corrente filosófica em particular. Sou eclético, não só em Filosofia, como em qualquer assunto. Não consigo me identificar com nenhuma linha de pensamento em sua totalidade. Do mesmo modo que com as ideologias políticas e as concepções psicológicas, sociológicas e econômicas. Cada caso, eu analiso e concordo ou discordo com as propostas explicativas das diferentes correntes. Ou então elaboro a minha própria, diferente de todas que existem. O que eu externo em tudo o que digo é o que pude concluir de tudo o que examinei a respeito. Pode ser que coincida com o que alguém já disse também. Mas não fico preocupado em saber que linha estou seguindo. Isso é um horror para os acadêmicos.

Professor, desconsiderando a sua provável falta de tempo, o senhor estaria aberto a aulas particulares? Ou algo que se aproxime disso.‎

Infelizmente não tenho a menor possibilidade. Minhas incumbências do trabalho me tomam de 10 a 12 horas por dia. O resto é para o meu lazer, que inclui responder a essas perguntas. Mas sem compromisso.

Michio Kaku tem credibilidade cientifica?‎

Ele é um pesquisador da Teoria das Cordas que entende bem do assunto, bem como de espaços de dimensões múltiplas. Além do mais tem uma didática muito boa. Mas ele peca em apresentar as hipóteses da Teoria das Cordas e das Branas, bem como dos Universos Múltiplos como se fosse algo já estabelecido como certo, o que não é. Para um leigo ele passa a impressão de que a ciência já considera que essas hipóteses sejam teorias consolidadas, o que não são. Para se inteirar delas, desde que se saiba que não são confirmadas ainda, e podem nunca vir a ser, as explanações dele são ótimas.

considerações sobre o modelo helical de órbitas ao redor do sol?‎

Em relação a um referencial centrado no Sol, os planetas o orbitam em elipses, perturbadas pelas interações interplanetárias e pelos momentos superiores de distribuição da massa do Sol, especialmente o de quadrupolo, devido ao achatamento solar nos polos. Isso provoca a precessão dos periélios (a precessão da eclítica é devida à precessão da rotação da Terra em torno de si, ocasionada pelo torque que a atração da Lua e do Sol fazem, devido a provocar o atrito das marés - isso tem a ver com o gradiente do campo gravitacional do Sol e da Lua ao longo do diâmetro da Terra e não com o valor do campo no centro da Terra). O Sistema Solar, como um todo (considerado em seu centro de massa, que praticamente coincide com o centro do Sol), orbita o centro da Galáxia, como também oscila para cima e para baixo em relação ao plano galático, como os cavalinhos de um carrossel. O plano da eclítica (isto é, o plano das órbitas dos planetas) faz um ângulo de 27° com o plano galático, de modo que, em relação a um referencial supergalático (parado em relação a um conjunto de galáxias e quasares muito distantes) a trajetória dos planetas é, realmente, helicoidal, mas com uma hélice generalizada, de plano geratriz oblíquo em relação ao eixo do cilindro diretriz.Todavia não é verdade que o Sol vai "puxando" os planetas. Eles sempre estão todos orbitando um mesmo plano em relação ao Sol. Esse plano vai se deslocando pela Galáxia com sua inclinação mantida constante em relação ao referencial supergalático. Outro aspecto interessante é que, à medida que todas as estrelas da galáxia giram, esse giro não é nem rígido (todas juntas), nem do tipo que fazem os planetas, pois a massa gravitante para cada estrela é toda a massa galática contida numa esfera centrada no centro da galáxia. E essa massa vai aumentando com a distância ao centro. Isso faz com que as estrela se aglomerem em braços espirais com espaços rarefeitos entre eles, mas vão passando de um braço para outro, enquanto a Galáxia gira. Veja isto:
http://www.astro.iag.usp.br/~gastao/BracoEspiral/index.html .

Qual é a diferença entre Ateu e Judeu?

Judeu pode ter dois significados. O primeiro é uma pessoa adepta da religião judaica. O segundo é uma pessoa pertencente à etnia judaica. Grande parte dos judeus por religião são judeus por etnia e vice-versa. Mas há judeus por religião que não são judeus por etnia e vice-versa. Ateu já é uma pessoa que considere que não existam deuses. Um judeu por etnia pode ser ateu.

É normal eu sentir muita, mais muita raiva? Eu sinto raiva de tudo professor, qualquer coisa, até a mais banal me deixa um pimentão de tanta raiva.‎

Acho que não é normal não. Você deveria procurar um psicólogo e fazer uma terapia para conter esse impulso. Isso é prejudicial para sua vida e para as outras pessoas que se relacionam com você.

Como vc explica as pessoas endemoniadas?

Ou é uma encenação ou é um caso de histeria, um problema psíquico.

Professor, NUNCA vi você recomendar livros de Matemáticas que abordam o conteúdo do E. Fundamental, e, ao meu ver, a maior parte do pessoal que são defasados em Matemática no E. Médio, é devido a falta de conhecimento da Matemática básica. Quais livros você recomenda para uma pessoa com os ditos cuj‎

Não estou inteirado dos livros de Matemática para o Ensino Fundamental. Quando eu estudei, na década de 60 do século passado, segui a coleção do Ary Quintella, que era muito boa. Depois eu só lecionei para o Ensino Médio e o Superior, perdendo contato com a bibliografia do Ensino Fundamental.

Você se incomoda de alguém te chamar como você, tu ou mesmo pelo nome?‎

Não só não me incomodo como prefiro. Assim fui educado por meus pais que o foram pelos pais deles. Todos profundamente liberais. Em nossa família todos se chamam de "você". Nada de "senhor". Meu paí, inclusive, como filho de mãe portuguesa e pai austríaco, que falava o português de Portugal, sempre usava o "tu". Minha concepção é de que todas as pessoas são essencialmente iguais, de modo que o tratamento de senhor, que indica uma diferença hierárquica, não procede. Nem para juízes e delegados.

Se seu filho dissesse a você que não gostaria de ter filhos, mesmo que fosse adulto, você insistira para ter um neto?

Não. Isso é uma questão de competência dele e de sua companheira. Não acho válido se fazer nenhuma pressão a respeito. Inclusive porque os filhos não são responsabilidade dos avós, exceto no caso da falta os pais.

Meu irmão de 9 anos disse a mim que pra ele Deus não existe. Todos na nossa familia somos cristãos, gostaria de saber se é normal isso acontece por parte da criança, ou seria influencia de alguém? Uma criança pode parar, refletir e chegar a essa resposta sem ninguém pra ajudá-la?

Acho difícil. Normalmente a criança acompanha a orientação religiosa da família até uma idade em que seja capaz de contestar. Isso costuma acontecer na adolescência, mas a adolescência, psicologicamente, é uma fase que se inicia em idades variáveis conforme a pessoa. O mais provável é que alguém, de fato, tenha influenciado o seu irmão. Nesse caso, é bom se discutir serenamente com ele as razões que o levam a não acreditar em Deus e contra-argumentar com as razões que a família admite para se acreditar em Deus. Mas é preciso estar aberto para que a criança convença a família de sua concepção pela impropriedade da crença em Deus. Se se detectar qual a pessoa que influenciou a criança, seria bom, também, estabelecer uma discussão franca e aberta com ela a respeito de suas convicções, ou para que ela convença a família ou para que a família a convença de estar errada. Nunca, porém, pode-se admitir a crença de modo dogmático ou impositivo e sim por convencimento, com base em argumentos de grande plausibilidade. Aliás á isso que os padres, pastores, rabinos, mulás, gurus, monges e sacerdotes de todas as religiões precisariam fazer em suas preleções em seus templos, para que todos os fiéis o fossem porque ficam convencidos da propriedade de sua fé, por argumentos sólidos, firmes e bem embasados, com lógica e plena validade epistêmica. Certamente que uma fé gratuita não tem nenhuma garantia de validade, uma vez que há fés genuínas e sinceras em uma multiplicidade de crenças completamente distintas, que se contradizem em muitos pontos. E a verdade, certamente, tem que ser única.

Ernesto, não tendo conhecimento matemáticos suficientes para entender as equações de Maxwell, como entender a unificação da eletricidade e magnetismo em eletromagnetismo?‎

Por meio dos fenômenos da indução de um campo elétrico pela variação de um fluxo de campo magnético, descoberta por Faraday e por meio da indução de um campo magnético pela variação de um fluxo de campo elétrico, descoberta, também, por Maxwell. São ocorrências experimentais que permitem entender a ligação da eletricidade com o magnetismo. Podem ser vistas em um laboratório. Esse assunto está meridianamente bem explicado, em nível de Ensino Médio, no quarto volume da coleção de Física do PSSC, editada pela EDART e, preliminarmente, pela Universidade de Brasília.

http://pt.euronews.com/2014/03/18/big-bang-a-prova-que-faltava/ Professor, é realmente verdade?

Comprovações do Big Bang já as havia. O que agora foi verificado foi a expansão inflacionária que se havia proposto que tivesse acontecido logo após o Big Bang. A teoria do Big Bang não foi proposta por Alan Guth. Este propôs a hipótese da Inflação Cósmica. Além disso o Big Bang não é "só" uma teoria. É uma teoria porque é comprovada e já o era. Seria "só" se fosse uma hipótese. Teoria é uma hipótese comprovada.

Se a tensão nas extremidades (nas duas) de uma corda forem iguais , a tensão em cada ponto possui esse valor , mas se a tensão em cada extremidade tem valores diferentes , como faço para calcular a tensão em cada ponto da corda ?‎

Nesse caso tem-se que considerar a massa da corda e o seu peso, calculando a tensão em algum ponto intermediário pela segunda lei de Newton, que diz que a soma vetorial das forças é igual à massa multiplicada pela aceleração vetorial. A massa é proporcional ao comprimento. Daí se tira a tensão que uma parte da corda exerce sobre a outra. Ao se desprezar a massa da corda, necessariamente se tem que considerar a tensão sempre igual em toda a corda.

Professor, se Deus não existe e a gente "veio" dos macacos, quem os criou?‎

Nenhum ser foi "criado". Todos os seres vivos evoluíram uns dos outros desde o primeiro que surgiu espontaneamente a partir da matéria inanimada. A matéria inanimada, por sua vez, também surgiu espontaneamente sem causa, propósito e de que provir, quando o Universo começou. Aliás, sem ter do que provir também é o que consideram os que acham que o Universo surgiu em um ato de criação de alguma entidade extrínseca a ele.

acredita em ''poliamor''?

Não é uma questão de acreditar e sim de constatar. De fato, há casos em que uma pessoa ama, sincera e profundamente, a mais de uma outra pessoa e sofre muito por ter que reprimir um dos amores para ficar com o outro, porque a sociedade nem as outras pessoas amadas admitem o compartilhamento amoroso em triângulos, quadrados ou outras formações poliamorosas de forma consentida e acolhida com satisfação, alegria e felicidade pelas pessoas envolvidas, sejam de que sexo forem.

Por que eclipses solares são mais raros que os lunares?‎

Porque a área projetada da Lua, que tem que ocultar o Sol de nossa visão é 16 vezes menor do que a área projetada da Terra, que tem que ocultar o Sol visto da Lua, para que haja eclipse da Lua. Então há muitas situações orbitais, na Lua Nova, que não provoca eclipse do Sol, mas que se fosse a mesma situação, na Lua Cheia, provocaria eclipse da Lua. Há trajetos da Lua à frente do Sol nos quais ela passa raspando e não o oculta, mas se fosse da Terra em frente ao Sol, vista da Lua, o ocultaria.

O que você acha de um aluno que faz muitas perguntas?‎

Gosto muito. Porque isso me leva a esclarecer, como professor, o que eu penso que ficou entendido e não ficou. Alguns não entendem mas têm vergonha de perguntar. Os que não têm vergonha prestam um excelente serviço aos que têm, pois levantam as dúvidas. Algo que para o professor possa parecer óbvio, para a turma pode não ser. Os alunos perguntadores não permitem que essa suposição errônea seja mantida e, assim, a turma não leva o prejuízo de não entender. Por outro lado, também gosto muito dos alunos que entendem tudo e não se contentam apenas com o que está sendo ensinado e querem ir além. Mesmo que não vá ser objeto de avaliação, acho muito bom isso, pois abre a mente de todos para os desdobramentos do que se está estudando. Realmente esses são os alunos que eu gosto. Não gosto de quem se contenta em saber só o que vai cair na prova.

Como aceitar o fato de um amigo estar supostamente apaixonado por " você" no caso eu.‎

Não vejo problema. Se você não quer namorá-lo, é só dizer a ele. Não precisa cortar a amizade e nem ele precisa deixar de te amar. É só aceitar a situação tal qual se apresenta. Isto é, um amor unilateral. Isso não é bom, mas é melhor que não amar. Mas ele tem que entender que você não o ama e aceitar isso com tranquilidade, sabendo que pode ser só seu amigo. Quem sabe ele não ama a outra pessoa também, que queira namorá-lo. Isso não implica que ele tenha que deixar de te amar. Amor, quanto mais, melhor.

O senhor resolve suas coisas devagar para saírem quase perfeitas ou por ser característica da sua pessoa, ter cautela com tudo?‎

Devagar, para mim, é uma filosofia de vida. Ou seja, acho que tudo deve ser feito devagar, exceto em casos críticos, para que se possa fruir o prazer de fazer o que se está fazendo. Com isso, é claro, se pode fazer tudo da melhor maneira possível e eu sou um perfeccionista. Não aceito o mais ou menos e nem o bom. Tem que ser ótimo ou excelente. Tanto é que não me preocupo com a eficiência de nada e sim com a eficácia. Nem com o custo. Não me importa o trabalho que dê e nem o tempo que consuma. Tem que ficar excelente. E, geralmente, tudo que é melhor é mais complexo e mais difícil de ser feito. A simplicidade não é melhor do que a complexidade. Claro que, para o mesmo resultado, um processo mais simples é preferível. Mas se o simples ficar bom e o complexo ficar ótimo, eu escolho o complexo. E a complexidade exige dedicação e cuidado que não se obtém com a pressa.

Nós somos macacos?

Sim, se chamarmos de macaco a todos os espécimes da ordem primata, em especial da sobordem dos haplorrinos, infraordem dos simiformes, na qual se enquadra a família dos hominídeos, que inclui os gorilas, bonobos, orangotangos e todas as espécies de humanos que já existiram, inclusive a nossa. Alguns, contudo, excluem da denominação de macacos as espécies da subtribo hominina, que são os diversos "homos", que houve, dos quais só existe, hoje, a nossa espécie.

Ser humano é a criatura mais frágil da natureza?

Não. É, até, das mais robustas. A maior parte dos seres vivos, tanto em quantidade de espécies quanto em quantidade de indivíduos é mais frágil do que o ser humano.

Professor, estou num dilema. Curso Engenharia, área onde a preocupação com o conhecimento é focada na técnica, prática e mercado. Queria ser matemático, cientista, sem ligar se o conhecimento seria usado na prática, mas o curso de Matemática não oferece alta perspectiva financeira. Pode-se ser os 2?‎

Não acho que a perspectiva financeira de um matemático seja menor. Fazendo mestrado e doutorado você pode ser um pesquisador de uma Universidade e ganhar um salário compensador. Claro que você tem que ser um ótimo matemático, para passar no concurso na frente dos outros e obter o lugar. De qualquer modo é possível fazer engenharia e matemática também, um após o outro. Especialmente porque matemática existe em turno noturno e você pode trabalhar de dia e estudar à noite.

o que acha desta pesquisa http://veja.abril.com.br/noticia/educacao/revista-forbes-aponta-as-graduacoes-mais-valiosas ?‎

Não acho que o valor de um curso se meça pelo salário que seus graduados poderão conseguir. O que importa é o impacto que seus graduados podem fazer para melhorar o mundo, mesmo que não ganhem tanto. Assim, por exemplo, para mim, os cursos de licenciatura são os mais importantes de todos. Claro que medicina é importante, mas não pelo salário dos médicos e sim por poder melhorar a saúde da população. O curso de jornalismo, também, é muito importante, pois os jornalistas são uma espécie de consciência do povo. Pelo menos deveriam ser. Não é fácil fazer um levantamento do impacto de cada curso na melhoria do mundo, mas acho que isso precisava ser feito, para não ficar apenas em opiniões.

precisa ser feita alguma faculdade pra ser escritor(a)?‎

Claro que não! Mas o curso de Letras deveria fornecer condições para se ser escritor. Só que não o faz. Para mim é uma grande falha. Os cursos de belas artes preparam a pessoa para ser pintor ou escultor, os de música para ser executor e compositor. Por que os de letras não formam escritores? É claro que vai depender do talento pessoal, como os pintores, escultores e compositores. Mas quem faz curso de jornalismo aprende mais a escrever do que quem faz curso de letras.

numbers exist?‎

Certamente que existem. A questão é saber o que significa existir. Existir é ser parte da realidade. E há mais de uma categoria de realidades. Há a realidade física ou natural, das entidades substanciais da natureza, feitas de matéria, campo e radiação, e suas estruturas e ocorrências, no espaço e no tempo. Mas há também a realidade dos conceitos. Estes só existem em mentes que os concebam. Mas não são apenas subjetivos, pois é possível se estabelecer um consenso entre subjetividades que confere objetividade aos conceitos, isto é, eles passam a ser independentes do sujeito pensante. Todavia não existem se não houver, pelo menos, um sujeito que os conceba em sua mente. É o caso dos números, das figuras geométricas e outros entes matemáticos. Pode, também, haver alguma realidade não natural e não abstrata, como seriam os espíritos. Ou seja, eles não seriam naturais, nem tampouco meros conceitos, mas existiriam mesmo sem mentes que os concebessem. Nesse caso, como no caso das realidades físicas, há que se verificar sua existência fora das mentes. Não me consta que tal verificação tenha logrado sucesso, até o momento. Portanto espíritos (incluindo deuses) não passam de abstrações mesmo. Como os números. Mas, como abstrações, existem.

Professor, gosto muito de Física, todavia pretendo cursar Engenharia de Software primeiramente. Gostaria de ser um Astrofísico também, mas não sei se seria o correto para mim.

Por que não seria correto? Um engenheiro pode ser também físico e astrofísico. Nada impede estudar vários assuntos. Mesmo não correlacionados. Como Engenharia e Direito ou História, ou Música. A questão é só que se despenderá mais tempo. Isso vai depender do fato de já se ter uma fonte de renda ou não. Mas é possível estudar e trabalhar. Cursei Matemática à noite, trabalhando de dia. É só dormir pouco e não ver nunca televisão.

Acabo de finalmente ingressar no curso de física e quero ser um bom físico. Não necessariamente ter ótimas notas. Quero chegar no final do curso pensando como um cientista. Gostaria de recomendações do senhor, principalmente no que se refere a hábitos e gerenciamento de tempo. Obrigado!‎

O fundamental é ter um profundo deslumbramento pela natureza e desejar compreendê-la. Ser possuidor de uma curiosidade insaciável, que não esmorece com as dificuldades. Ter disposição para meter a cara e pegar o boi pelo chifre, porque o caminho é árduo e não se pode descurar de palmilhar todos os passos, inclusive os mais áridos e pedregosos, que envolvem muita matemática. Mas não se pode ter um espírito tecnicista. Há que se ter abertura para a intuição. Os hábitos de estudo são os mesmos do nível médio. Assistir as aulas ativamente não consentindo em sair de nenhuma sem ter entendido tudo. Para isso pode chatear o professor à vontade e pagar todos os micos. Depois de um descanso e uma refeição, estudar a aula no mesmo dia, antes do sono. Isso é ESSENCIAL. O bom é estudar como se estivesse preparando uma aula para ensinar aquilo. Quanto ao gerenciamento do tempo, o importante não é tanto planejar, mas aproveitar toda brecha sem a menor preguiça. É mais uma atitude de não procrastinar. Bem como não perder tempo com futilidades. Mas há que haver tempo para o lazer também. Mesmo para namorar. Mas é preciso evitar ver televisão e ficar muito na internet. Um pouco pode, bem como pesquisar.

Considero algo perfeitamente alcançável por meio da evolução da civilização. Inclusive vejo que tem sido a tendência milenar e secular da humanidade, de tal modo que fatalmente será alcançado em alguns milênios. Mas esse tempo pode ser abreviados para poucos milênios ou alguns séculos se se fizer um esforço positivo de conscientização pela educação. É preciso entender, contudo, que a anarquia, que é o suprassumo da civilização, não pode ser alcançada por revolução nenhuma, mas só por evolução, pois revolução é totalmente contrária aos conceitos anarquistas. Não se trata de utopia, mas de uma possibilidade real.

Gosto mais do Thomas. Também aprecio o velho e bom Granville. No curso de Matemática que fiz se adotou o Willie Alfredo Maurer, da Edgard Blücher. Também gostei muito.

Acredita no anarquismo como algo realmente possível ou apenas como uma ideologia utópica?‎

Considero algo perfeitamente alcançável por meio da evolução da civilização. Inclusive vejo que tem sido a tendência milenar e secular da humanidade, de tal modo que fatalmente será alcançado em alguns milênios. Mas esse tempo pode ser abreviados para poucos milênios ou alguns séculos se se fizer um esforço positivo de conscientização pela educação. É preciso entender, contudo, que a anarquia, que é o suprassumo da civilização, não pode ser alcançada por revolução nenhuma, mas só por evolução, pois revolução é totalmente contrária aos conceitos anarquistas. Não se trata de utopia, mas de uma possibilidade real.

A matemática é como uma obra de arte. Ao se debruçar em equações passa-se pelo corpo uma sentimento de prazer. Receio que isso não se passe em todos. É como ouvir música clássica resolvendo problemas. Como se os números e letras fossem uma harmonia só, e ao resolvê-los, fossemos os maestros.‎

Concordo plenamente. Mas entendo que haja quem não se extasie com a matemática, como não se extasia com a música clássica. A propósito, há muita relação estrutural entre as duas. Que o digam os pitagóricos.

O senhor acha que uma pessoa que é homossexual atualmente ou que "sempre foi", nasce homossexual ou essa "opçao" é desenvolvida e percebida ao longo do tempo?‎

A homossexualidade é uma orientação sexual nata, ao lado da assexualidade, bissexualidade e heterossexualidade, que pode ser consentida e vivenciada ou reprimida. Também pode ser uma opção escolhida em desacordo com a orientação nata, por livre vontade, perfeitamente válida. Nesse caso, sendo uma opção e não uma orientação natural, pode ser chamada de homossexualismo. Todavia há quem considere que a denominação homossexualismo não deva ser aplicada. No meu entendimento pode, quando for opção livre, distinta da orientação natural. Mas não pode para o caso de ser orientação natural. O mesmo se aplica às outras possibilidades. Por exemplo uma pessoa homossexual por orientação natural pode praticar o heterossexualismo por opção voluntária. Do mesmo modo que uma pessoa heterossexual por orientação natural pode praticar o assexualismo por opção voluntária. É o que acontece com os sacerdotes católicos que fazem voto de castidade e o cumprem. Certamente que a homossexualidade, como as demais orientações sexuais naturais, se manifesta plenamente a partir da puberdade. No entanto é possível perceber sinais dessas orientações ainda na infância, em alguns casos.

Professor oque acha sobre a legalização da maconha? Não acha que ela só é proibida no Brasil porque o governo não lucra com isso? Outras substâncias como bebida e cigarro são mais prejudiciais a saúde e são licitas‎

Não só por isso que é proibida. Há uma pressão norte-americana. E há o interesse de muita gente que lucra com a venda ilegal da maconha. Essas pessoas, inclusive, dominam vários políticos, quando não são eles mesmos.

Ser ruim em matemática implica ser desinteligente? Ou, menos inteligente que os que são bons em matemática? Existe menosprezo entre pessoas boas para com as ruins em matemática?

Claro que não ser bom em matemática não significa não ser inteligente. A inteligência possui vários aspectos, dentre os quais o matemático, mas não só. Há o linguístico, o musical, o plástico, a espacial, o cinestésico, o interpessoal e vários outros. Uma pessoa pode ter maior ou menor inteligência em alguns ou em outros. Ou em todos ou em nenhum. Isso varia. O teste de QI mede apenas a inteligência lógico-matemática, a linguística e a espacial, que se configuram no conjunto de fatores acadêmicos da inteligência e que, portanto, são suscetíveis ao nível de escolaridade. Mas, mesmo dentre eles, pode haver sobressalência em um ou outro. Acontece, também, o "fator G", ou seja, uma inteligência destacada em todos os aspectos. Todavia, não é só a inteligência que mede o valor de uma pessoa. Mais importante do que a inteligência são o caráter e os sentimentos. Mesmo uma maior inteligência não é garantia de maior sucesso adaptativo ao mundo. Além de que, o que se considera como sucesso, em termos, geralmente, de prosperidade, não é o valor mais importante da vida. Quanto a haver menosprezo de pessoas boas em matemática em relação às que não sejam boas, isso pode haver, mas, caso haja, configura-se uma situação em que a inteligência maior não é acompanhada de uma elevação de caráter, de sentimentos e de sabedoria.

É interessante notar que os críticos de Deus sempre imaginam que criariam um mundo perfeito. Mas, um mundo onde tudo fosse arbitrariamente "bom" não teria liberdade, nem emoção. Viver significa correr riscos

Não sei o que a ideia de um mundo ideal tenha a ver com Deus. Todavia, o judaísmo e o cristianismo consideram que, ao criar o mundo, Deus o fizera como um paraíso. Só depois é que houve a expulsão de Adão e Eva. Claro que isso é lenda. Só estou dizendo que os crentes em Deus também acolhem a noção de um mundo paradisíaco. O ideal anárquico de uma sociedade boa, justa, harmônica, fraterna, pacífica, ordeira, próspera, alegre e feliz não é uma arbitrariedade e nem reprime liberdade nenhuma. É assim por consenso e livre escolha de todos. E isso não tolhe nenhuma emoção, exceto o medo e a insegurança. Viver sem correr riscos é, exatamente, viver mais plenamente ainda.

A matemática é apenas um reles acessório para técnicos e engenheiros e professores, economistas, comerciantes e sistema bancário e Industria. Quem cria a mudança no Mundo, são os filosofos e pensadores, e revolucionários que são péssimos em matemática.‎ .

Claro que não! Que ideia preconceituosa! Matemática não é reles coisa nenhuma. É um poderoso instrumento de compreensão da realidade que possibilita, inclusive, o controle da natureza. Engenheiros, técnicos, professores, comerciantes, economistas, médicos, cientistas são tão importantes para a humanidade quanto filósofos e revolucionários. Além disso, não é verdade que, pelo menos filósofos sejam péssimos em matemática. Muitos filósofos foram e são matemáticos. E não há nada que implique em que revolucionários sejam péssimos em matemática.

Quais os grandes homens realmente bons em matemática? Genghis Kan? Cesar? Napoleão? os inventores na maior parte nem bons alunos eles eram. A rebeldia é indispensavel para libertar o pensamento, os matematicos estão presos num curral mental de números e formulas bobas.‎ .

Gengis Khan, César, Napoleão, Alexandre absolutamente não foram grandes homens., Foram péssimos exemplos para a humanidade. Quem deu grandes colaborações para o bem da humanidade foram, de fato, filósofos, artistas e cientistas, dentre os quais matemáticos também. No pórtico da Academia de Platão estava escrito "Que ninguém exceto os geómetras entrem aqui". Não me consta que nenhuma filósofo menosprezava a matemática, mesmo não sendo matemático. Mas Descartes, Leibniz, Pascal, Russell, Wittgenstein e muitos outros filósofos foram matemáticos também. Não é verdade que matemáticos não sejam rebeldes e nem que estejam em nenhum curral mental, Isso é um preconceito sem tamanho. Além do mais, as fórmulas matemáticas não são bobas de modo nenhum. Sem a matemática a ciência não progrediria. Não só a ciência mas a engenharia, a arquitetura, a economia, a contabilidade, a administração, a geografia e a maior parte das atividades humanas. Acho essa sua concepção da matemática completamente deplorável e indicadora de uma mente antolhada.

Usar Matemática para explicar tudo é um erro crasso.

Certamente. Mas isso não se faz, em ciência. Todavia também é um erro crasso não se valer da matemática para levantar possibilidades a serem exploradas. Como é o caso de túneis de minhoca, buracos brancos, universos paralelos, velocidades superluminais, super-cordas e outras hipóteses não confirmadas, mas previstas teoricamente, que os cientistas buscam confirmação. O mesmo vale para a possibilidade de vida extra-terrestre. Além do mais, tudo que seja passível de um modelamento explicativo matemático é muito mais válido, pois permite o controle dos fenômenos e a previsão de resultados a serem verificados com precisão. Enquanto as previsões forem confirmadas, o modelo continua válido, até que se encontrem discrepâncias que forcem a sua correção ou abandono.

Acha que pode existir mais de um universo? Ou talvez que nosso universo seja apenas parte de algum sistema maior?

Pode, mas não há nada que indique que haja. Trata-se de uma conjectura teórica, advinda da extensão matemática das soluções cosmológicas e das soluções dos modelos de teoria quântica de campos para as partículas elementares da natureza, por exemplo, a hipótese das super-cordas. Todavia, tal suposição só pode ser aceita como válida se houver alguma evidência de tal fato ou, pelo menos, um indício muito forte. Isso ainda não foi detectado.

Há como reporuzir e demonstrar esse espaço-tempo ou não se sabe se ele de fato existe como é dito? Pode falar mais sobre ele? http://ask.fm/wolfedler/answer/110489342749#_=_‎

Não há como produzir espaço e tempo. O que existe é o que estamos imersos nele e não há como escapar. O que se pode, e já se fez, é verificar se as ocorrências gravitacionais são corretamente descritas pelo modelo de interação newtoniana, em um espaço-tempo plano, ou pelo modelo de inércia em um espaço-tempo curvo. Os dois modelos levam a previsões diferente em casos limites. Vários testes já foram feitos. Os mais famosos, historicamente, foram o do desvio da luz das estrelas em torno do Sol, observado em eclipses do Sol e o da medida da precessão do periélio de Mercúrio. Eles confirmaram a validade do modelo de espaço-tempo curvo, previsto pela teoria da Relatividade Geral. Atualmente, por exemplo, os aparelhos de GPS. que informam a latitude, longitude e altitude de um lugar a partir de sua detecção por um conjunto de satélites, têm que se valer de correções fornecidas pela Relatividade Geral para a dilatação e contração do tempo de viagem dos sinais quando a favor e contra a gravidade da Terra. Esse fenômeno não é previsto pelo eletromagnetismo clássico, mas se não for levado em consideração não fornece os resultados precisos para a localização do receptor.

Professor já que o senhor não aceita a teoria que Deus criou nosso mundo. Como explica? NA SUA VISÃO sem teorias malucas

Não existe nenhuma teoria maluca a respeito do surgimento do Universo. Isso ainda é uma fato sujeito a estudo. A teoria do Big Bang (que não é maluca, de modo nenhum), não se refere ao surgimento do Universo e sim ao fenômeno de sua súbita expansão. O que se considera mais plausível é que seu surgimento tenha se dado imediatamente antes, ou seja, que ele já tenha surgido expandindo-se. Mas pode ser que esse conteúdo que começou a expandir já existisse, ou desde sempre imperturbado ou como resultado final da evolução de um Universo anterior. As mais recentes medidas da aceleração da expansão e da densidade de massa e energia indicam que o Universo não seja cíclico, tendo tido um momento inicial. Esse é um assunto fascinante para ser investigado, mas não é fácil, uma vez que é muito difícil se obter informação sobre os instantes iniciais do Universo. Acho que tudo surgiu nesse momento inicial, não havendo nada antes. Nem mesmo espaço vazio. E nem "antes", pois o tempo surgiu ali também.

Por que " massa atrai massa" ? qual o íntimo disso ? Todos falam da gravidade, força gravitacional.. mas por que ocorre ? Não sei bem se o senhor entendeu minha pergunta.‎

Não se sabe porque ocorre. Em verdade o que acontece é que a massa provoca encurvamento do espaço-tempo, de modo que os outros corpos se movem inercialmente nesse espaço tempo curvo, sob ausência de interações, como se estivessem sofrendo uma interação gravitacional. Mas não se sabe porque as massas e a energia provocam encurvamento do espaço tempo. Nem se sabe porque um corpo livre de interações move-se ao longo das trajetórias inerciais do espaço tempo, denominadas geodésicas.

A força do pensamento seria uma energia?

Não. Não existe "força do pensamento". O pensamento só consegue criar disposição para agir sem esmorecimento. Mas são as ações que promovem alterações no mundo. O pensamento, sem ação, não é capaz de nada, exceto produzir outros pensamentos ou algum sentimento.

http://ask.fm/wolfedler/answer/110489001757 e esta velocidade lateral provém de que?‎

Do modo como foram formados, já com ela. Já expliquei isso aqui: As estrelas são formadas por acreção de gás e poeira do espaço, a partir de uma concentração aleatória localizada, que passa a atrair o que existe em volta. Mas essa matéria não cai exatamente radialmente, havendo componentes laterais de velocidade que provocam uma resultante de rotação à estrela. No começo, antes mesmo que ela inicie suas reações nucleares, o efeito centrífugo cria um disco em torno, a partir do qual, também por acreção em torno de alguma concentração aleatória, se formam os planetas. Estes, por sua vez, também não agregam gás e poeira de modo completamente radial, então se formam girando. Ao se formarem, como foram provenientes do material expulso pela estrela devido a seu giro, já se formam orbitando a estrela. Raramente pode ser que algum escape e depois seja capturado por outra estrela.

Não se pode afirmar números e quantidades de planetas habitados, a Matemática apenas conta e registra o que já existe, ela por si própria não determina as ações do que existe ou por algum motivo se transformou ou transformará. Matématica é estatistica, ela apenas numera o que já existe.

Não. Estatística não é só levantamento de dados. É projeção também. Aliás, isso é o mais interessante na estatística. São os levantamentos estatísticos que fornecem a maior parte dos dados astronômicos, astrofísicos e cosmológicos. Isso inclui, até, as distâncias entre os astros, seu brilho e várias outras informações. Dentre elas a possibilidade de existência de vida extraplanetária.

Prof Ernesto, porque os planetas giram ao redor do Sol? Eu sei que é por causa da gravidade e tal, mas não entendo porque eles não colidem com o Sol, em vez de ficar girando ao redor, sendo que a gravidade do Sol é muito grande..‎

Um corpo em órbita é um corpo em queda que também tem uma velocidade lateral. Ele está sempre caindo, mas nunca chega ao Sol (ou à Terra) porque está avançando, enquanto cai. Imagine uma pedra lançada para o lado, de uma janela. Ela cai a certa distância. Se for lançada de um andar mais algo ou com mais velocidade, cai mais longe. Se for lançada de centenas de quilômetros de altura com dezenas de milhares de quilômetros por hora não chegará nunca ao chão, porque, sendo a Terra esférica, o chão vai saindo de baixo dela. Aí ela fica sempre querendo ir para o chão e nunca o alcança. No caso dos planetas em relação ao Sol é a mesma coisa. A Terra, por exemplo, foi formada a 150 milhões de quilômetros do Sol, com uma velocidade lateral de 108 mil km/h. Então ela nunca cai no Sol. A gravidade do Sol, onde a terra está, fornece, exatamente, a aceleração centrípeta para que ela fique numa órbita aproximadamente circular em torno do Sol. O mesmo se dá com os outros planetas.

Você acha justo uma pessoa boa ir para o mesmo lugar que um estuprador assassino após a morte?

Nenhuma pessoa vai para lugar nenhum após a morte. Com a morte a pessoa deixa de existir. Isso não é justo nem injusto. É fato.

O sr. acha que no Brasil há na realidade uma democracia propriamente dita?‎

Sim. Mesmo eivada de grandes falhas operacionais e, até, no meu entendimento, estruturais, a democracia brasileira está estabelecida organicamente na constituição e nas leis e vem funcionando, aos trancos e barrancos. Como já disse, o que se precisa é fazer cumprir tudo o que está previsto, especialmente na condução do processo político, sem os vieses coronelistas e corruptos que ainda prevalecem. Mas eu acho que seria melhor se fossemos parlamentaristas, sem coligações partidárias, sem voto proporcional, sem voto obrigatório e com candidatura sem partido.

Possibilidades não são fatos, portanto se quer mesmo ser levado a sério, deveria não pender sua forma de pensar para o reino da fantasia, deveria somente passar informações concretas, sem iludir o grande número de seguidores que tem.

Possibilidades não são fantasias. São possibilidades. Não estou iludindo ninguém. Jamais afirmei que não existem extra-terrestre e nem que existem. Podem haver sim. Porque não? Mas, se houver, serão bem raros, como as possibilidades indicam. Isso não é nenhuma fantasia. São conjecturas completamente plausíveis. Contatos com extra-terrestres é que não há possibilidade de haver, no meu entendimento, justamente por causa das grandes distâncias envolvidas. Isso eu já disse e repeti inúmeras vezes. Os propalados avistamentos, ou são fraudes, ou são equívocos. Então, tanto erram que acha que o Universo está lotado de extra-terrestres que visitam a Terra, quanto os que acham que não seja possível haver vida inteligente a não ser aqui na Terra. Estes, muitas vezes, se baseiam em argumentos religiosos. Esses sim, são "Maria vai com as outras", porque se fiam em suas escrituras que não tem nenhuma garantia de serem verdadeiras. Tanto não têm que existem múltiplas religiões. Claro que não podem ser todas verdadeiras, pois apresentam propostas divergentes. Como saber qual é a certa? Pela fé? Mas em todas elas há pessoas de fé sincera no que pregam. O mais razoável é considerar que todas sejam erradas.

Testemunho baseado em quê? em falácias mirabolantes e fantasiosas? eles viram a vida em outros planetas para testemuinharem o que viram? e mesmo que vissem, haveria a possibilidade de estarem vendo ilusões da mente, vocês agnosticos são fáceis de enganar, francamente viu?

Caríssimo. Você não está entendendo. Não se pode garantir nem que haja nem que não haja vida inteligente em nenhum outro lugar fora da Terra. O que se pesquisa são as possibilidades de haver planetas com condições propícias para que vida inteligente possa ter se desenvolvido. Isso é algo perfeitamente possível de ser feito. Mas não garante que haja nem que não haja. Assim procede a ciência. As religiões, por outro lado, garantem suas assertivas. Sem fundamento, contudo.

Não se sabe se existe vida em outros planetas a não ser que se entre em contato com eles, o que seria impossível, uma vez que nem mesmo contato através de ondas de rádio poderia existir de tão longe que estariam, caso existirem mesmo, e se tiverem tecnologia e interesse, em contactar.

Eu não falei que se sabe. Eu falei de possibilidades. Não se sabe que tem nem que não se tem. É o mesmo caso a respeito de deuses e espíritos. Então se tem que basear em possibilidades teóricas. E isso é o que foi feito. Tanto é possível que há um projeto, o SETI, com o qual colaboro e você pode também, para detectar possíveis inteligências extra-terrestres, o que ainda não se deu. Se eu participo do projeto é porque considero que a possibilidade existe, mesmo que remota. Procure por "SETI on line" na internet.

Puras conjecturas, meras suposições de pesquisadores fantasiosos, quimeras desses falaciosos charlatões da pseudociência, que querem vender livrinhos bobos para leitores mal informados, e que possuem dúbia formação.

Essa é a sua opinião, da qual discordo. Leia o livro que você verá que não é nada bobo. Além do mais, não me considero um leitor mal informado e nem possuidor de dúbia formação. Aliás, essa é uma área em que, coincidentemente, me especializei em minha pós-graduação, ou seja, Cosmologia. Mesmo que o tema, em particular, não tenha sido o objeto de minha pesquisa de tese, faz parte do conjunto de assuntos a que me dediquei a estudar, ou seja, astronomia e astrofísica.

E você é Maria vai com as outras? só porque os pesquisadores falaram você acredita neles? não tem opiniões próprias não? os religiosos também afirmam que Deus existe, e nem por isso se deve acreditar neles. Você é um concordino.

Para mim, o testemunho de pesquisadores é muito mais aceitável do que a palavra das ditas escrituras sagradas. A ciência tem mecanismos de contrafação que não são aceitos no seio das religiões. Os cientistas sempre que propõem algo, isso é objeto de questionamento e verificação por seus pares antes que seja aceito. Nas religiões não se permite contestar suas assertivas para verificar sua veritabilidade.

Angel Acuna me mandou uma pergunta, em inglês, ao longo de múltiplas postagens, dizendo que, se se encontra, por acaso, em uma praia de uma ilha deserta, uma inscrição em algum idioma, presume-se, corretamente, que tenha sido obra de alguma inteligência. Então, como o código do DNA é um tipo de insc‎

A analogia não se aplica. São situações completamente diferentes. O código do DNA não foi colocado pronto em cada ser vivo atualmente existente. Ele foi surgindo gradativamente, a partir de sistemas replicantes extremamente simples que foram evoluindo ao longo de mais de 3 bilhões de anos até que surgiram as primeiras archeas e bactérias. E isso ainda demorou centenas de milhões de anos até a explosão cambriana, quando a vida procariota pluricelular se desenvolveu. Até chegar ao DNA dos seres vivos atuais a evolução se deu através de milhões de gerações. Isso pode, sim, acontecer por acaso.

3 Planetas habitados por galáxia? de onde tirou isso? você esteve lá pra saber? não se pode afirmar nem que sim, nem que não, ao supor que exista vida fora da Terra, está sendo igual os religiosos que afirmam sem ter certeza de nada. Ridiculo!

Este levantamento foi feito por pesquisadores com base em probabilidades e consta do livro "Sós no Universo" de Brownlee e Ward. Não é um valor determinado e sim provável. Refere-se a vida inteligente. Esse livro é, realmente, bem interessante e bem embasado. Jamais disse que tenho certeza disso. Aliás eu sempre digo que não possuo certezas a respeito de quase nada. Mas possuo convicções. Este caso, contudo, não é de convicção e sim de projeção. Não considero que seja ridículo conjecturar sobre possibilidades com base em fundamentações estabelecidas com boa razoabilidade. Isso não é o que acontece com as religiões nem com certas crenças em extraterrestres, por exemplo. Estas sim, são como as religiosas.

Como o Brasil pode ser livre? Eu não acredito muito que seja, já que a verdade sempre nos é oculta e que para buscar uma notícia verossímil temos de procurar por ela, enfim, um país onde se censura muitas coisas, que se mencionar é uma enorme lista e liberdade de expressão como? Se falar demais.


 Pode até ser preso, realmente não acredito nesse aspecto de país "livre", professor. E se eu estiver errada, por favor, corrija-me.
O problema é conjuntural e não estrutural. O Brasil é estruturalmente livre, mas não conjunturalmente. O que é preciso é fazer com que também o seja conjunturalmente. E isso é algo que cabe a todas as pessoas que vivem nele. Todo mundo tem que fazer valer a constituição e a lei e agir para que aqueles que não as cumprem, especialmente políticos, juízes, policiais e empresários, mas mesmo trabalhadores assalariados, sejam exemplarmente punidos. Não se pode omitir. O voto é uma grande arma nesse sentido. Outra, importantíssima, é o esclarecimento. Outra, ainda, é a denúncia. O problema é que a maior parte dos bons é medrosa e os maus não. Não se conseguirá impedir os maus de fazerem maldades enquanto se temê-los.

Professor, as leis físicas tem fim? Por exemplo, os modelos atomicos que evoluiram ao longo do tempo. há algo que ainda está por evoluir bastante? e o que não pode mais evoluir?‎

Você está se referindo a um prazo de validade? A princípio, quando se formula uma lei da natureza, seja física, química, biológica, geológica, astronômica ou outra, se pretende que seja definitiva. Mas isso nunca pode ser garantido, pois sempre é possível que novas descobertas venham invalidar o que as leis vigentes estabelecem. É importante ressaltar que as leis da natureza não são prescritivas e sim descritivas. Elas não dizem como a natureza deve agir, mas sim como ela age. E ela age à revelia das leis.

Salvar vidas não é melhor que compor músicas? Adoro música, mas é um fato que o mundo conseguiria viver sem músicos, mas não sem médicos. Se alguém faz uma coisa indispensável, ele é melhor do que quem não faz. E eu tava falando de competição nos esportes, a competição do esporte não causa mal.

Não acho que salvar vidas seja melhor do que compor músicas. É mais necessário, mas não melhor. A competição nos esportes pode causar mal sim. Só não causa se for algo levado "na esportiva", ou seja, sem o compromisso de ganhar e sem se importar em perder. Mas, muitas vezes, as pessoas levam o esporte muito a sério, como desporto, e isso é mal mesmo.

Professor, por que é tão difícil para algumas pessoas entenderem a Teoria da Evolução? Algumas delas, apesar de toda a capacidade cognitiva, simplesmente a negam e a ridicularizam. Veja: https://www.youtube.com/watch?v=LBkSFCgf8SA Vídeo que exemplifica o que muitos falam a respeito da teoria.‎

Porque as pessoas não se aprofundam em seu estudo e captam uma visão distorcida da evolução. Muitas vezes isso é propositalmente feito, como no caso do vídeo, por quem tenha uma oposição ideológica, de fundo religioso, à teoria da evolução. Para começar é preciso que as pessoas entendam que as explicações científicas têm que ser buscadas sem nenhuma concepção prévia, fornecida por nenhuma religião. O que se busca é a verdade acerca da realidade. E as concepções religiosas não são baseadas em nenhuma busca dessas, mas em interpretações doxistas (http://pt.wikipedia.org/wiki/Doxa) que foram construídas ao longo do tempo, desde a pré-história e, depois, consignadas nas ditas "escrituras sagradas". O que é preciso é se inteirar de forma correta do que significa a evolução das espécies e como ela se processa, que não é nada do que o professor do vídeo explana. Mas, muita gente nem quer saber, e já se posiciona contra por preconceito.

Sabe dizer como surgiu a elite social?‎

A elite social possui bens materiais e culturais porque tem recursos para obtê-los, recursos esses que costumam ser obtidos por herança. Mas, primordialmente, esses recurso advieram do poder que, em última instância, se fundamenta na força. Então é verdade que a riqueza, com exceções, no fundo, é um roubo, mesmo que o rico não seja ladrão, mas, historicamente, em geral, seus ancestrais, há milênios, enriqueceram por terem dominado, escravizado, extorquido e usurpado o que era dos mais fracos. Claro que há quem se enriqueça pelo suor de seu trabalho, mas isso é bem raro. Daí a necessidade de se conceder a todos, pela sociedade, as mesmas oportunidades de conseguir desabrochar seus talentos e capacidades. Evidentemente que todos não serão iguais, pois uns as aproveitarão e outros não. Mas a igualdade de oportunidades é a única concepção social justa que se pode ter. Esse é o grande mal da concepção direitista de sociedade, a qual considera que as desigualdades sejam justas e, mais ainda, necessárias para a harmonia social. Mentira! Essa harmonia só o é para os ricos. Para os pobres não é harmonia nenhuma. Numa sociedade justa, fraterna, igualitária e verdadeiramente harmônica pode haver elites naturais em razão da desigualdade natural que faz com que uns se sobressaiam por seus dotes de nascença e por sua dedicação e esforço. Mas isso tem que ser algo a que todo mundo, sem exceção, tenha o direito de conseguir ser, não sendo obstaculado por nenhuma condição de nascença

Tenho uma grande curiosidade sobre Deja Vu, o que diria sobre isso? Seria apenas um lapso de memória que a memória recente dá uma apagada por segundo de momento, fazendo ter a sensação que já viveu isto? Queria entender esses famosos deja vu‎

É isso mesmo. O cérebro acaba de registrar uma percepção e, por um lapso de funcionamento, a rememora como se estivesse armazenada há muito tempo.

Professor, vc acha que seria ignorancia alguem dizer que algo é impossivel baseando-se que a tecnologia nunca chegaria ao ponto de realizar o feito? porque eu acho que a tecnologia poderia aumentar a ponto de realizar tal feito, como por exemplo as armas que evoluiram ao longo de 2 milenios


Depende. Há coisas impossíveis de serem alcançadas, não importa o nível da tecnologia. Por exemplo um motor que forneça energia sem consumir nada. Ou se mover com velocidade maior do que a da luz. Isso não depende de tecnologia, pois vai contra leis físicas estabelecidas.

Você não gosta de competição, mas você considera ela uma coisa ERRADA? Tem gente que se sente feliz competindo (mesmo quando perdem), e fazem isso sem prejudicar ninguém. Se ajudar é melhor que competir, então não devemos competir? Ser médico é melhor que ser músico, então não devemos ser músicos?

Quem gostar de competir que compita. Mas eu não acho que seja uma coisa boa. E acho que é errada mesmo, se a competição não for meramente uma diversão, mas uma luta para, de fato, vencer às custas de causar prejuízo ao derrotado. Acho que o ideal, mesmo, é não competir e sim ajudar a todo mundo a melhorar e superar suas deficiências, alcançando o sucesso. Um mundo ideal é um mundo em que todos, sem exceção, sejam cultos, saudáveis e prósperos. Isso pode ser alcançado se toda competição for substituída por colaboração. Essa comparação de médicos com músicos foi infeliz, pois médicos não são melhores do que músicos.

as pessoas tem alma?‎

Não, se se entender por alma uma entidade incorpórea unida mas distinta do corpo, que dele se separaria na morte, continuando a existir como depositário da memória, da personalidade, dos sentimentos e do psiquismo da pessoa. Se se chamar de alma apenas a mente, que é uma ocorrência que se dá no cérebro e órgãos associados, então pode-se dizer que tem. Mas essa alma não é uma entidade substancial não material com existência própria. É só uma ocorrência. No caso, um tipo de epifenômeno do corpo, que não sobrevive a ele.

Professor Pier menciona que é durante o sono que é fixado o que você estudo.Mas por exemplo,se eu estudo a matéria do dia e depois vou dormir, então aquela informação será gravada ou tem que ser, necessariamente, uma 'noite de sono'?‎

Tem que ser a "noite de sono". Uma soneca depois do almoço não promove isso. Mas, para que a fixação aconteça é preciso que o conteúdo a ser memorizado seja reforçado no mesmo dia, antes que se durma. Ou que tenha sido acompanhado de uma carga emocional. Só que o reforço tem que acontecer em outro momento, não em seguida, senão o cérebro não entende como uma revisita ao assunto. É preciso dar um intervalo de umas horas entre o primeiro contato e o estudo. O ideal é ter aulas pela manhã, nas quais se toma conhecimento dos conhecimentos e habilidades e estudar à tarde, esquematizando o conhecimento e treinando as habilidades, para que o entendimento e a compreensão fiquem estabelecidos e, então, no sono, a fixação e a aprendizagem sejam concretizadas.

acerto

terça-feira, 15 de abril de 2014

Em quanto tempo mais ou menos, tendo um QI de 130 em média, fazendo exercícios de neuróbica, se consiga algo próximo de 150 de QI ou até mesmo 150 redondo?

Não conheço experimentos que revelem esse tempo. Mas acho que isso pode acontecer em menos de cinco anos. É mais difícil do que emagrecer. A perseverança tem que ser muito grande e a preguiça nula. É preciso adquirir uma verdadeira atração pela complicação da vida e pelo enfrentamento de desafios cognitivos bem puxados. É como treinar para ser um atleta. Só que um atleta mental. Há quem já tenha essa facilidade de nascença. Mas ela pode ser conquistada, sem a menor dúvida. Não facilmente, contudo

Ernesto, poderia me explicar de forma simplificada o que seria um quasar? e qual sua relação com os buracos negros?‎

Atualmente considera-se que os quasares são núcleos ativos de galáxias distantes nos quais há um buraco negro supermassivo que esteja sugando a galáxia para dentro de si. Enquanto isso acontece o material galáctico se ioniza e se acelera espiralando para o buraco negro. Essas cargas aceleradas é que emitem a potente radiação que se percebe. O nome quasar significa objeto quase estelar, que é observado como se fosse uma estrela mas é extremamente mais longínquo e brilhante.

Ernesto, li no seu blog e você possui habilidade em diversas áreas. O que fazer para me desenvolver em diversas áreas do conhecimento (musica, literatura, exatas, medicina)? Já me disseram que quem é bom em tudo, não é bom em nada; que o melhor é focar em uma coisa para ser o melhor nela e não em va

Discordo completamente dessa concepção. Uma pessoa pode, perfeitamente, ser boa em vários assuntos. Isso se chama "polimatismo". Havendo múltiplos interesses, não há porque abandonar uns se se fixar num só. Mesmo que se fixando num só se possa ser muito melhor ainda nele, não compensa o fato de não se dedicar a outros assuntos de que se gosta. E não é preciso ser bom em tudo. Mas é muito gostoso ter uma multiplicidade de interesses e se dedicar a eles com empenho, se se é capaz. E não se deve pensar na utilidade prática do que se faz. Faz-se porque se gosta e não porque se precisa ou para obter dividendos. Se se consegue manter a vida fazendo o que se gosta, isso é ótimo. Se não se importa em ser rico, pode-se dispender um bom tempo com outras ocupações com as quais não se ganha dinheiro e, até, se gasta.

A turquia não pode entrar na União Européia,porque ainda não respeita os direitos humanos e mal os de liberdade religiosa .E ia encher toda a Europa de imigrantes muçulmanos. iria ficar horrível‎

Já existem muitos turcos na Europa. Não acho que a Xenofobia seja algo bom. Todavia o respeito aos direitos humanos e à liberdade religiosa é um fator que tem que ser atendido mesmo. Se o governo turco quer entrar para a União Européia ele pode promover isso. Alíás a Turquia já foi um exemplo de país muçulmano liberal e regrediu. Isso significa que a sociedade turca, como a iraniana, é capaz de aceitar esse tipo de liberdade. O que impede é o poder das elites preconceituosas e ciosas de sua superioridade idiota.

Gosta de futebol ou de qualquer outro esporte?

Não. Nunca gostei, nunca joguei, nunca assisti. Não acho graça nenhuma. Especialmente os que usam bola e têm times. O que eu tenho grande aversão é o espírito de competição que acompanha o esporte. A atividade física e a confraternização me agradam. Mas não o fato de ter-se que vencer. Acho que tudo na vida tem que ser feito de modo que uns ajudem aos outros, para que superem suas limitações. Sou contra qualquer tipo de competição. Mas aprecio, por exemplo, caminhadas, escaladas, corridas, ou outras atividades em que não se precise vencer ninguém. Que sejam feitas para fruir o prazer de fazê-las. Do mesmo modo não gosto dessas "Olímpiadas" de Física, Matemática ou outras. Não acho bom as pessoas curtirem o fato de serem melhores do que as outras. Concurso de beleza, então, para mim é uma abominação. Na escola eu acho que todos os bons alunos devem ajudar aos que não são bons para que se tornem bons. Na vida também. Desse modo, todo mundo sendo igualmente bom no que faz, a desigualdade social será cada vez menor. Claro que existem desigualdades pessoais, tanto de força, agilidade, resistência, inteligência, disposição, empenho e tudo o mais. Mas é preciso que sejam minimizadas. E as próprias pessoas é que têm que ajudar e incentivar, umas às outras, para que melhorem e sejam mais capazes. E não agir de modo a que se leve os outros a serem derrotados e prejudicados.

Como você define buraco negro e distorção do espaço-tempo?

Buraco negro é uma região do espaço dentro da qual a curvatura do espaço-tempo (campo gravitacional) é tão intensa que não permite que nada atravesse a sua fronteira de dentro para fora (mesmo a radiação de Hawking não a atravessa, mas é gerada nela). Normalmente é produzido pelo colapso gravitacional de uma estrela que esgotou sua disponibilidade de energia nuclear e explodiu em uma supernova de forma tal que o caroço remanescente ficou com uma massa maior do que 2,7 massas solares (limite de Oppeheimer-Volkoff). Nesse caso, nem a degenerescência do gás de elétrons nem a degenerescência dos nêutrons consegue impedir a gravidade de comprimir a matéria até que ela se reduza a uma unica partícula elementar com a massa do caroço da estrela todo. Isso faz com que essa massa ocupe uma região de raio menor do que o raio de Schwarzschild, que posiciona o "horizonte de eventos" e forme, assim, o buraco negro.
Uma distorção no espaço tempo é uma súbita variação local de sua curvatura, causada, por exemplo, por uma súbita contração de matéria, como acontece na formação de um buraco negro a partir de uma super nova. Isso ocasiona uma quebra da suavidade da curvatura, quebra essa que se propaga, como uma onde na água a partir de onde uma pedra caiu. Isso é uma onda gravitacional, cuja quantização é denominada graviton.

O que pensa sobre a entrada da Turquia na União Européia? Tem cabimento, mesmo com a parte européia desse país correspondendo a apenas 5-10% do território? Minha mãe é turca mas não tem opinião formada sobre isso ainda, aí queríamos saber a opinião do senhor se possível...‎

Acho que tem cabimento sim e acho bom. Mesmo a Turquia sendo muçulmana, ela tem uma história de ligação muito grande com a Europa, pois foi sede de muitas cidades gregas e, mais tarde, do Império Romano do Oriente (Bizantino), que teve uma grande influência na civilização dos povos eslavos. Portanto a Turquia tem vínculos com a Europa maiores do que a fração européia do seu território.

O senhor recebe um número muito alto de perguntas?

Muito mais do que eu consigo responder. Sem contar as que eu recusei, já respondi 8728 e tem 10391 na fila de espera, em, mais ou menos, um ano. Isto dá uma média de umas 50 por dia, mas, às vezes, chegam mais de 100. Só dou conta de responder a média de 20 por dia, pois tem dia que nem acho hora de acessar a internet. Recebo uma média de 10 likes por resposta, mas tem umas que já estão chegando a quase 1000 likes. Veja isto em: ask.fm/wolfedler/best. Não me preocupo com os likes. Apenas menciono para mostrar que, afinal de contas, mesmo sendo uma pessoa mais velha em meio a maioria de jovens daqui, eles apreciam o que digo. E foi para isso mesmo que eu entrei aqui. Para difundir conhecimento e provocar reflexão. No formspring eu respondi mais de 12 mil, em três anos. Meu blog mais antigo já tem quase dez anos e 4575 postagens. O outro, mais novo, tem uns 6 anos e 14284 postagens, mas lá eu deixo cópia do que digo aqui e dizia no formspring. Também participei de muitas comunidades de discussão no orkut, desde a sua fundação, em 2005.

Professor, supondo que não flutuássemos no espaço, ao sair da órbita da terra, "onde" cairíamos (supondo também que não houvesse nenhum obstáculos - temperaturas altas, baixas, buracos negros, asteroides, etc -?‎

Em lugar nenhum. Permaneceríamos vagando pelo espaço, Como a nave Voyager, ao se sair da órbita da Terra, entra-se em órbita circunsolar, que pode ser parabólica e se afastar para sempre do Sol. Para ser espiral teria que haver propulsão. Saindo do âmbito do Sistema Solar, entra-se em órbita galáctica. Se se escapar da galáxia, fica-se vagando pelo espaço sideral até se aproximar de outra galáxia e passar a orbitá-la. Isso acontece com muitas estrelas que escapam da galáxia.

Professor, por quê certo dia eu fiz uma pergunta sobre gravitons e ondas gravitacionais, e tu me ignorou, você não tem muito conhecimento do assunto?

Não me lembro de ter visto essa pergunta. Chegam várias dezenas por dia e eu não dou conta de responder. Muitas vão ficando para trás. Quanto às ondas gravitacionais elas são perturbações na curvatura do espaço tempo, produzidas por alguma alteração de distribuição de massa e energia. Isso acontece em grandes cataclismos cósmicos e astrofísicos. Essas perturbações se propagam através do espaço, ao longo do tempo, como se fosse um tipo de radiação gravitacional. Gráviton é a quantização dessa radiação, como fóton é a quantização de onda eletromagnética. Mas ainda não se detectaram gravitons. Recentemente se detectou uma indicação indireta, por meio da polarização da radiação de fundo do Universo (que é eletromagnética), cuja causa pode ser atribuída a ondas gravitacionais geradas pela súbita expansão inflacionária do Universo em seus instantes iniciais.

Qual a diferença entre um material isolante de um material condutor quanto a sua constituição atômica?‎

Os isolantes são feitos de moléculas covalentes ou coordenovalentes. Os condutores são os que possuem ligação metálica ou iônica, estes se fundidos ou em solução. Os compostos iônicos sólidos também são isolantes.

Exato, eu acho mesmo que se pode fazer duas coisas. Mas acho que não por não se poder dar conta de todas, mas sim porque a sociedade tem a tendência a dizer que se deve perseguir uma área, ou escolher entre duas profissões, por exemplo.‎

Isso é bobeira. Nunca ouvi ninguém dizer isso. E mesmo que dissessem, o que importa? Não se pode pautar a vida pela opinião dos outros ou porque vão falar mal. Faça-se o que se considera válido e os outros que se danem. Desde que, é claro, não se faça nada que prejudique ninguém.

O fluxo de tempo é como a mecânica cinemática de quadros dum filme?

Não se sabe ao certo se o tempo tem um fluxo contínuo ou qqueno. Há uma proposta do físico italiano Caldirola a esse respeito. Veja isto:http://en.wikipedia.org/wiki/Chrononuantizado, isto é, há um intervalo mínimo do qual todos os outros sejam múltiplos inteiros. O tempo, assim, fluiria aos saltos. Se assim o for, esse quantum de tempo seria muito pe

Você não acha triste que a sociedade capitalista diz que devemos ter carreira ou seguir uma só profissão? Eu gostaria de fazer várias coisas

Nunca ouvi dizer que seja preciso se dedicar a uma única profissão. Penso que as pessoas fazem isso por que não dão conta de se dedicar a mais de uma. Mas quem der conta, não há impedimento nenhum de assim proceder. Qualquer um pode ser um médico e um fazendeiro, um professor e um músico, um aviador e um pintor ou, mesmo, três ou mais ocupações. Inclusive pode ganhar dinheiro nelas todas.

Há algo que um buraco negro não consiga "sugar"?‎

Qualquer coisa que não entre em seu "horizonte de eventos" não será sugada por ele. O que adentrar, seja o que for, atingirá o seu caroço e não poderá mais sair. Esse horizonte é uma esfera imaginária, com centro no caroço do buraco negro e raio igual a 2GM/c² em que G é a constante de gravitação universal, M a massa do buraco negro e c a velocidade da luz. 2G/c² vale 1,48E-30km/kg. No caso do Sol, esse raio vale cerca de 2,94 km, isto é, se o Sol se comprimisse a um volume de raio menor do que isso, seria um buraco negro com esse "raio de Schwarzschild" e, então, se concentraria em um caroço imensamente pequeno. Tudo o que se aproximasse dele menos do que esse raio seria tragado. Todavia o Sol não tem massa suficiente para que seu colapso o transformasse em um buraco negro.

Como ocorre a interação entre corpos carregados? Existe algum esquema específico para isso?

O campo elétrico é o mediador dessa interação. Em verdade pode-se considerar que cargas positivas sejam fontes e as negativas sorvedouros de campo elétrico. Os campos atuam sobre suas fontes e sorvedouros de modo a buscar a redução global do campo. Então as cargas de sinais opostos se atraem e as de mesmo sinal se repelem, sendo a intensidade da atração e da repulsão proporcional à densidade do fluxo do campo que vai de uma à outra. Essa densidade é diretamente proporcional ao módulo da carga e inversamente proporcional ao quadrado da distância, exatamente porque à medida que a distância aumento a área abrangida pelo fluxo aumenta com o seu quadrado, fazendo a densidade (fluxo por área) diminuir com o quadrado, ou seja, variar com o inverso do quadrado. No Universo como um todo o fluxo global é nulo, o que significa que o total de cargas positivas e negativas é o mesmo, tando uma carga líquida total zero. Sempre que surge alguma carga de qualquer sinal é preciso que surja uma carga oposta ao mesmo tempo, em algum lugar. Normalmente isso acontece por separação de cargas de algo previamente com carga zero. É o que acontece com a produção de pares de matéria e antimatéria a partir de fótons.

Quero ser professor universitário, mas tenho medo de não levar jeito... talvez eu nao tenha didática, oratória.. essas coisas. O senhor pode-me dizer algo quanto a isso ?‎

Mas isso tudo se aprende e se treina. Para isso existem disciplinas nas Universidades: Didática do Ensino Superior, Retórica, Redação, Dialética e Argumentação, Lógica. Tudo isso é importante para se saber convencer ao apresentar um conteúdo que se pretenda que os alunos aprendam, não importa a natureza do conteúdo. O importante é sentir uma vocação para disseminar conhecimentos e habilidades. E ter um grande deslumbramento pelo assunto que se quer lecionar. Tem que ser fissurado mesmo.

Quantas línguas estrangeiras é preciso saber para cursar um doutorado?‎

Há um exame de proficiência em duas línguas. Normalmente uma é o inglês, pois é a língua da maior parte das comunicações científicas internacionais. A outra depende do lugar em que se está fazendo o curso. Geralmente há um leque de opções. As mais usadas são francês, alemão, russo ou japonês.

Com o decorrer do tempo a tendência da nossa sociedade é de piorar ou de melhorar? Por quê?‎

Pelo que se pode depreender da tendência secular e milenar da humanidade, mesmo com idas e vindas, a humanidade está melhorando cada vez mais em quase todos os aspectos, como expectativa de vida, saúde, nível de instrução, prosperidade, paz, solidariedade, liberdade, tolerância, consciência política, social, ecológica, e tudo o mais. Portanto, a tendência é de continuar melhorando. Inclusive há uma aceleração nessa melhoria, ou seja, a derivada segunda é positiva. A curva é uma logística e ainda não atingimos o ponto de inflexão.

Um orbital (eletrônico) é uma entidade de fato ou é apenas uma área imaginada pra facilitar nossa visualização do átomo?

Há duas interpretações. Uma é de que o orbital seja um conceito matemático que representa o campo de densidade de probabilidade de que o elétron, suposto puntiforme, possa ser encontrado, em torno do átomo. Outra é que, de fato, o elétron seja espalhado em torno do átomo, constituindo um campo de matéria não puntiforme, quando sob ação do campo elétrico e magnético do núcleo (e dos outros elétrons). Sou favorável a esta última interpretação. Mas não há um experimento que permita, por enquanto, decidir entre elas.

As características de personalidade podem ser corrigidas ou existem traços que são vitalícios? Como arrogância, competitividade? (mais as negativas)‎

O que é inato são as tendências. Mas elas podem ser reorientadas pela educação. Certamente é custoso. Mas as tendências não são determinantes. Podem ser controladas.

Mas você não acha que deveriam existir mais evidências históricas a respeito da existência de Jesus?

Esse Jesus divino e miraculoso é uma invenção. Logo não há mesmo evidência histórica dele. Mas essa invenção deve ter sido baseada em alguém que existiu e propunha uma reforma do judaísmo (um cismático, isto é, um herege).

Por que você acredita na existência do Jesus Histórico, mesmo não havendo evidências de sua existência?

Penso que houve uma pessoa que corresponde àquela de que se fala. Mas a maior parte do que se atribui a ela é invenção, especialmente os milagres, a natureza divina e a ressurreição.

★ Sempre foi seu sonho ser Matemático Ou não?

No adolescência eu queria ser engenheiro mecânico ou arquiteto. Depois fui mudando meu interesse para Física, Matemática e Filosofia. Meu sonho era ser físico, pois assim eu reuniria os três ramos. Mas acabei fazendo Matemática, pois em Barbacena, onde eu morava, não tinha Física e meu pai não tinha como me manter em Belo Horizonte, onde havia. Comecei a trabalhar já no primeiro ano da faculdade, para pagar meus estudos. Mas fui ser professor de Física, que estudava sozinho o que ia além da Física Geral do curso de Matemática. Depois, já como professor universitário, fiz meu mestrado em Física. A Filosofia sempre estudei por conta própria, desde 13 anos de idade.

Por que ao olhar as estrelas a noite, a olho nu, elas parecem piscar? É porque passam na frente delas alguma coisa?

Porque a atmosfera possui flutuações de índice de refração, causadas pelas flutuações de densidade, pressão e temperatura do ar, não só com uma dependência da altitude, mas também de modo flutuante e aleatório, devido a ventos etc. Isso faz com que os raios de luz vindos das estrelas, que abrangem um pincel extremamente estreito, sofram variações de direção, ora acumulando, ora dispersando. O que se vê passando na frente podem ser nuvens mesmo, às vezes bem pequenas. Ou, simplesmente, a refração atmosférica desvia o raio da direção do olho momentaneamente e a estrela deixa de ser vista por pequenos instantes. Quando a estrela é mais brilhante, isso é menos acentuado, como também ocorre com os planetas Mercúrio, Vênus, Marte, Júpiter e Saturno. Urano e Netuno já se apresentam como estrelas pequeninas.

Professor como inicio uma tebela verdade? o que por em p,q,r,s? o resto eu sei‎

Se você tiver uma proposição composta por quatro proposições elementares p, q, r e s, considerando que cada uma pode ser verdadeira ou falsa, você tem que fazer uma tabela com 2^4 = 16 linhas, nas quais você vai distribuir todas as possibilidades para p, q, r e s, ou seja:
VVVV
VVVF
VVFV
VVFF
VFVV
VFVF
VFFV
VFFF
FVVV
FVVF
FVFV
FVFF
FFVV
FFVF
FFFV
FFFF
Depois você vai compondo o resto da tabela, dependendo dos operadores lógicos que ocorrerem na proposição composta, começando de dentro para fora, segundo a tabela verdade de cada tipo de operador.

Por que você considera que a vida seja um fenômeno raro se a Terra é repleta dela?

Porque a Terra é só um entre centenas de bilhões de planetas nesta galáxia que é uma entre centenas de bilhões de galáxias no universo observável. E que a existência de vida nos planetas requer condições especiais que poucos preenchem, especialmente vida inteligente que, se houver, será no máximo em uns três planetas por galáxia.

O que um anarquista pensa sobre o marxismo?

Não concordo. Acho que o verdadeiro comunismo tem que ser anarquista. O socialismo de estado e a ditadura do proletariado não são formas válidas de se atingir o comunismo. No socialismo de estado marxista todos são empregados de um único patrão, o estado. No comunismo não há empregados. Todos são sócios dos empreendimentos econômicos, que são comuns. Socialismo não é comunismo. E a combinação da política anarquista com a economia comunista é a situação ideal para a sociedade humana. O marxismo equivoca-se em considerar a ditadura do proletariado com a estatização da economia como um caminho viável para o atingimento do comunismo, que nunca aconteceu. Além do mais, também se equivocam os anarquistas que acham que alguma anarquia pode ser alcançada por meio de revoluções. As revoluções são intrinsecamente opostas ás concepções anarquistas. Revoluções exigem comando. Anarquismo não tem comando. Anarquia só pode ser alcançada por evolução e não por revolução.

Por que há tanto preconceito, na sua opinião, contra as pessoas que trabalham com ciência humanas, principalmente vindo das pessoas das exatas?‎

Porque, em geral, quem lida com ciências naturais (exatas e biológicas), lida com um conhecimento que se baseia em verificações e não em opiniões. E as ciências humanas, em geral, constroem suas assertivas com base em opiniões. Tanto que há divergentes escolas de pensamento nas várias ciências humanas. Isso não existe nas ciências naturais. Cada fenômeno admite, em cada momento, uma única explicação aceitável, até que se mostre incorreta e seja corrigida. Mas não há opiniões. Outro aspecto é que, em geral, quem se dedica às ciências naturais consegue, sem dificuldade, assimilar os conhecimentos das ciências humanas, mas a recíproca é mais raramente verdadeira. Há poucos entendidos em Antropologia que também entendem de Física Quântica. Mas há muitos entendidos em Física Quântica que entendem de História ou Filosofia, Sociologia ou outras ciências humanas. Daí a impressão que as ciências humanas passam de serem uma coisa mais fácil, que não requerem um nível de inteligência que as exatas e biológicas requerem para serem entendidas em profundidade. Isso é um equívoco. Para se entender em profundidade de uma ciência humana há que se ter muita inteligência também. Mas a impressão geral é essa. Inclusive, para cobrir essa lacuna, estou escrevendo um livro "Física para Filósofos", cujo andamento está sendo publicado na internet em http://pt.scribd.com/doc/80588829/FISICA-PARA-FILOSOFOS .

Professor, se não há nenhum lugar pra ir depois da morte, sendo o papel da religião é fundamental para dar um conforto as pessoas. Acha que sem ela, as pessoas entrariam em desespero e fariam o que bem entender, inclusive, o caos, ja que n ha um céu, e consequentemente n ha regras a se seguir?

Nada disso. Não é porque não existe céu nem inferno que se possa fazer o que se queira. Isso é niilismo. Somos uma espécie gregária e vivemos em sociedade. É preciso que haja restrição a comportamentos que comprometam a harmonia, a paz e a alegria dessa convivência. A própria sociedade tem que coibir os comportamentos perniciosos. Veja Isso:
http://www.ruckert.pro.br/blog/?page_id=1890
http://ateus.net/artigos/filosofia/conferencia-sobre-etica/
http://www.anda.jor.br/05/12/2009/etica-e-ateismo
http://adrianoped.blogspot.com.br/2007/04/atesmo-e-tica.html
http://www.paulopes.com.br/2012/04/pastor-elogia-ateismo-etico-e-critica.html

A fé, ela existe ou é algo inventado pelos religiosos?

Claro que existe! Fé é uma crença que não possua fundamentos em plausibilidades. Crença é a aceitação de proposições não evidentes ou não comprovadas, com base em indícios. A fé, geralmente, se reporta à crença em assertivas concernentes a realidades sobrenaturais. Mas também pode ser a crença na palavra de alguma pessoa. As religiões se alicerçam na fé de seus seguidores. Para que haja alguma religião é preciso que haja um grupo de pessoas que acreditem, mesmo sem evidências, provas e nem indícios, no que ela propõe que seja verdade a respeito da realidade, geralmente incluindo a existência de um domínio sobrenatural de entidades, como deuses, espíritos e similares. O que pode não existir não é a fé, mas o objeto da fé, isto é, aquilo em que se acredita. Mas a crença existe. O ideal, para mim, é que não se cresse em nada que não tivesse plausibilidade de ser verdade, mesmo que não seja evidente e nem comprovado. Infelizmente, contudo, a humanidade ainda possui uma grande parcela de pessoas que acreditam em proposições sem indícios de veracidade. Tanto que essas crenças são divergentes em vários grupamentos, ou seja, existe mais de uma religião. Se as proposições das religiões fossem evidentes e comprovadas, ou, pelo menos, tivessem fortes indícios de veritabilidade, haveria só uma religião no mundo, em todos os tempos.

É verdade que Fidel Castro e Che Guevara perseguiam homossexuais no regime cubano? Ou isso é só uma falácia?

Pode ser mentira. Mas não falácia, pois falácia não é uma afirmação falsa, mas um raciocínio inválido. Não sei se é verdade ou mentira.

Como é a sensação de estar morto?

Essa sensação não existe, pois sensações requerem a existência de um aparato sensitivo, que é um tipo de sistema nervoso em funcionamento. Percepções já requerem a existência de uma mente, que é um cérebro em funcionamento. Como um corpo morto não está em funcionamento, não possui sensações e nem percepções, como consciência, que é uma modalidade de percepção.

O ateísmo é mais "ligado" a concepção de que não existem deuses ou qualquer ser sobrenatural?‎

Ateísmo se refere à concepção de que não existam deuses. Geralmente também se estende à concepção de que não exista nenhum tipo de entidade sobrenatural, como almas, espíritos, anjos, demônios, gênios ou o que seja. Mas não necessariamente. O ateísmo pode ser a descrença na existência de deuses (fraco ou cético) ou a crença na inexistência de deuses (forte, dogmático ou gnóstico). O agnosticismo é a concepção de que não se pode saber se existem ou não deuses. O agnóstico pode ser ateu ou não

Ateísmo é uma "religião"?

Claro que não. Ateísmo é só a consideração de que não existam deuses. Religião envolve uma crença, um corpo doutrinário, geralmente uma escritura sagrada, templos, liturgias, sacerdotes e muitas outras características que, absolutamente, o ateísmo não tem. Pessoas ateias podem acreditar em espíritos e podem ter outras opiniões as mais diversas sobre os demais assuntos. O ateísmo não se configura em organização nenhuma, não tem nenhuma escritura sagrada, não tem um corpo doutrinário, não tem templos, não tem sacerdotes, nada que lhe configure como religião.

O video é lindo. só que tem uma parte meia sem lógica,pelo menos para mim. Na parte que fala,'' Nao mate os animais bla bla bla.. nos somos humanos,precisamos de carne...‎

A carne não é necessária. Mas pode-se comê-la. O que não se pode é ser cruel no abate e nem matar sem necessidade. Isso inclui matar para tirar a pele para fazer casacos. Ou esportivamente. De qualquer modo, é perfeitamente possível se viver sem comer carne.

Professor, o que diz sobre projeção astral/projeção da consciência?‎

Isso não existe. Trata-se de um tipo de alucinação, como a experiência pós-morte de quem voltou à vida em casos de morte clínica.

Ah, sim. Conheço isso. Mas digo uma viagem "mais profunda": Algo que afete diretamente a existência, como a passagem de uma semana. Gostaria de saber se esse conceito é matematicamente justificável e/ou plausível.‎

Mas não há limitação para esse avanço temporal. Pode ser de vários anos. Depende da velocidade de deslocamento relativo. O único problema é a competência tecnológica e a disponibilidade de energia para promover tais velocidades. Outra alternativa seria a dilatação temporal gravitacional. Mas essa, para ser significativa, requereria campos gravitacionais de tal intensidade que inviabilizariam a integridade estrutural de qualquer sistema físico, quanto mais de um organismo biológico. Fora essas duas possibilidades, não há outras. Isso não tem nada de matemático. É um fenômeno físico. A matemática só serve para calcular o valor do tempo. A ocorrência não é matemática.

Professor, uma viagem de um ser vivo ao futuro, em velocidade mais rápida do que a nossa distorção temporal atual, seria discutível?

Isso é perfeitamente possível e acontece. Qualquer um que viaje, mesmo em um avião, experimenta uma passagem mais lenta do tempo. Ou seja, quando ele chega ao destino, viveu menos tempo do que os que ficaram parados em relação à Terra. Isso significa que o momento em que ele chegou é futuro em relação ao tempo que ele viveu. Mas, para a velocidade de um avião, esse futuro está apenas menos de um milionésimo de segundo na frente, para uma viagem de circunavegação da Terra, a 1000 km/h. No entanto foi medido por meio da comparação de dois relógios atômicos, um que ficou na Terra e outro que embarcou em um avião e deu a volta em torno da Terra. Para que se vá razoavelmente ao futuro é preciso se mover a velocidades comparáveis à velocidade da luz.
http://www.fflch.usp.br/df/opessoa/TR-Exp-2-Avioes.pdf

Qual é o conceito de "nada" que havia "antes" do universo? Ele é eterno?

Nada não é algo. Portanto não pode ser eterno, pois não tem existência. Mesmo que fosse algo que tivesse existência, também não poderia ser eterno para o passado, pois antes de haver Universo não havia tempo e, logo, não há o que pudesse sempre ter existido. Note que isso é uma constatação fática, ou seja, fenomenológica. Não há impedimento lógico e nem metafísico para que algo não pudesse sempre ter existido. O dito "argumento Kalam" é falacioso. Nada é uma palavra que designa o conceito de inexistência do que quer que seja. Não existe "o nada", pois se existisse, seria algo. Não se pode dizer que o Universo surgiu "do nada", pois isso significaria que existia algo antes do Universo, "o nada", que teria se transformado em Universo. Não é isso. O Universo não surgiu de nada. Não havia nada para se transformar em Universo.

Niilismo seria acreditar que nada existe?

Nada disso. Niilismo não é nenhuma crença. É a concepção de que valores não existem, sejam éticos, estéticos ou de qualquer outra ordem. Que cada um aja de acordo com sua própria vontade, sem se importar com nenhuma convenção. Alguns pensam que o ateísmo seja uma concepção niilista, mas não é. Pode ser que alguém seja ateu e niilista. Geralmente os niilistas são ateus. Mas um ateu não precisa ser niilista. Não se pode, também, confundir a contestação de valores com o niilismo. Na contestação de valores não se considera que valores não devam haver, mas que alguns valores que existem são equivocados e devem ser revogados e substituídos por outros.

Ernesto, se existe uma maquina do tempo , seria impossível viajar pra antes da criação do universo?‎

Não há como se viajar para o passado, exceto ao longo de uma curva tipo tempo fechada, que pode existir em torno de um buraco negro supermassivo. Mas, nessa situação, nenhum sistema físico manterá a sua integridade, sendo totalmente desintegrado e transformado em plasma. Mas, mesmo nesse caso, não se poderia ir a um momento anterior ao surgimento do Universo, pois não existem momentos anteriores ao surgimento do Universo, uma vez que o tempo só passou a existir junto com o Universo.

O senhor disse certa vez que possui pouco interesse em economia. Trata-se de desapego mesmo ou faltaram oportunidades para conhecer?‎

Não é falta de oportunidade. É porque o assunto não me desperta interesse. Como vários outros, como Direito, Administração, Agricultura, Pecuária, Medicina, Propaganda, Turismo, Comércio e vários outros. Mas até que tenho um interesse filosófico pela economia. O que não me interessa, de modo nenhum, são as finanças.

como a materia venceu a anti materia ?

Tal assimetria surgiu a partir do momento, na expansão, em que a separação entre as partículas atingiu um valor maior do que o livre caminho médio delas, antes que decaíssem. No começo eram produzidos e aniquilados números iguais de partículas e antipartículas. Todavia há uma assimetria entre as meias vidas das partículas e suas correspondentes anti-partículas. Se elas decairem antes da aniquilação e se esse decaimento se der em tempos diferentes para cada uma, advirá uma diferença entre o número delas. Isso envolve quebras de simetrias de carga e paridade e falta de conservação dos números bariônicos e leptônicos. A teoria é complexa, tendo sido exposta pelo físico russo Andrei Shakarov. Vou dar uma estudada para ver se consigo expô-la de uma forma acessível. Isso aconteceu 380 mil anos após o Big Bang, quando a radiação se desacoplou da matéria e o Universo ficou transparente. Antes ele era todo como o interior de uma única estrela: completamente negro. Estrelas só são brilhantes da superfície para fora, de onde emitem luz visível. Com o desacoplamento, parou de haver produção e aniquilação de pares de matéria e antimatéria, tendo sobrado o resíduo atual de matéria e os fótons da radiação de fundo. Cada fóton corresponde a uma partícula de matéria ou antimatéria que existia. E existem um bilhão de fótons para cada partícula de matéria, atualmente. Isto significa que, no começo havia um bilhão de vezes mais partículas de matéria e antimatéria do que de matéria, atualmente.

A cultura feminista nos dias de hoje é o maior reservatório de paradoxos? Ex: Exaltam o útero e ao mesmo tempo lutam pela legitimidade do aborto; consideram a feminilidade uma construção cultural ao mesmo tempo em que cultuam uma suposta supremacia feminina... [+]

Existem duas concepções distintas: feminismo e femismo. O feminismo busca a igualdade dos gêneros, tanto em direitos quanto em deveres. O femismo é o reverso do machismo, isto é, uma concepção antagônica ao homem, que defende privilégios para as mulheres do mesmo modo que o machismo defende privilégios para o homem. O feminismo é uma postura corretíssima, pois não há como se considerar que homens e mulheres sejam distintos, exceto no aspecto estritamente ginecológico. Uma sociedade igual e justa tem que dar os mesmos direitos aos dois gêneros e exigir deles as mesmas responsabilidades. Sem nenhuma distinção. Feminismo é o movimento para que isso aconteça. Sou uma pessoa radicalmente feminista. Não admito que homens ganhem mais do que mulheres e nem que mulheres se aposentem com menos tempo de serviço do que homens. Igualdade total é como penso que tem que ser. Não há supremacia nem feminina nem masculina.

Oii boa tarde :) Eu queria te perguntar uma coisa. Assim,Você ja deixou Deus entrar no seu coraçao? Deixe as Leis da fisica,filosofia e entre outros e me diga. Você ja pensou em caminhar com ele? Para dizer que ele nao existe? (se for para ser mal educado,nao responda)‎

Eu já fui uma pessoa católica pia e fiel. Tanto que queria ser santo. Eu já tive fé. Deixei de ter em razão de meus estudos e reflexões. Por isso é que sei que acreditar em Deus não é sinal de burrice ou ignorância, mas, apenas, de falta de aprofundamento e reflexão a respeito do assunto. E pode ser que quem se aprofunde e reflita, conclua pela existência, diferentemente de mim. Mas eu concluí pela inexistência e pelo despropósito da fé.

Ernesto, existem eventos na Física Quântica que são por definição aleatórios? Digo, que não seja incapacidade de uma boa medição, ou variáveis ocultas? Se sim, quais?

Certamente que sim. A aleatoriedade é intrínseca ao comportamento da natureza. Mas variáveis ocultas é uma hipótese não confirmada. Não considero que existam. A totalidade dos fenômenos naturais envolve alguma aleatoriedade. Acontece que, caso o fenômeno englobe um grande número de eventos elementares. a probabilidade da ocorrência combinada fica tão concentrada em um valor que se pode falar de determinismo. É o que acontece em nível macroscópico. Em nível atômico e subatômico, contudo, a aleatoriedade é manifesta, como se observa nos decaimentos radioativos e nas emissões de radiação por átomos excitados.

Se você defende a poligamia, por que não ama outras mulheres além da sua?

O que eu defendo não é a prescrição da poligamia mas a possibilidade da poligamia. Ou seja, quem quiser ter mais de um marido ou mais de uma mulher, deve poder ter, se todos estiverem da acordo. E a sociedade tem que aceitar isso como normal. Não só para o casamento como também para o namoro. Amor não é algo exclusivista. Pode ser exclusivo, mas não é obrigado a ser. Há gente que ama a mais de uma outra pessoa e tem que fazer uma escolha de se relacionar só com uma delas. Por que? Por que não com todas? Evidentemente que isso tem que ser às claras, isto é, como o conhecimento e o consentimento dos envolvidos. Isso é imoral? É, pois a moral estabelece que os relacionamentos amorosos têm que ser biunívocos. Mas, no caso, é a moral que está errada, pois a pluralidade amorosa e relacional não fere a ética, em absoluto, se todos estiverem felizes com isso. Em que isso prejudicaria a qualquer um dos envolvidos? Só se as pessoas estiverem se relacionando amorosamente não por amor, mas por interesse econômico. Mas isso sim, é que não é ético, mesmo que a moral não proíba.

O que seria um "bom egoísta"?

Não considero que ser egoísta seja bondade nenhuma para ninguém. Ou seja: não existe "bom egoísta". Ser egoísta é um comportamento maléfico.

Professor,quero entrar na universidade ano que vem,mas to pensando no trote.Eu não suportaria deixar rasparem meu cabelo,adoro meu cabelo.Nesse caso,os alunos fariam outra coisa mais aceitável pra mim,ou eles não tem essa compreensão?‎

Isso depende muito de qual seja a Universidade. Em algumas, os veteranos não têm compreensão mesmo, de modo que você deve ir pensando na possibilidade de ficar sem o cabelo.

Como os cosmólogos e astrônomos sabem que o desvio para o vermelho foi ocasionado pelo movimento de fuga em alta velocidade do objeto, ou pela distorção ocasionada pela expansão do Universo durante o trajeto em que a luz foi emitida - em que havia maior distorção - até a luz chegar à Terra?!‎

Há três razões que poderiam explicar o desvio para a extremidade vermelha do espectro das raias espectrais de absorção ou de emissão de elementos presentes nas galáxias distantes. Uma é o efeito Doppler causado pelo movimento relativo de afastamento. Outra é a redução do comprimento de onda da luz emitida contra um campo gravitacional. A terceira é devido à expansão cosmológica do espaço. No caso de galáxias distantes, o desvio é tão grande que, para ser causado pelo campo gravitacional, todas as galáxias, à medida que fossem cada vez mais distantes, teriam que ter massas cada vez mais absurdamente grandes. E essa distribuição de massa teria que obedecer a uma mesma regra para todas as que estivesse à mesma distância de nós, o que nos colocaria em um lugar privilegiado, contrariando o Princípio de Copérnico ou da Mediocridade, de que todos os lugares do Universo sejam equivalentes. Da mesma forma, a explicação do efeito Doppler teria que considerar um movimento próprio das galáxias se afastando de nós a velocidades proporcionais a sua distância de nós, o que, também, violaria esse principio, além de exigir estupendas velocidades de afastamentos para grandes desvios espectrais. Por outro lado, a consideração de que o desvio espectral se deva ao inchamento do espaço não nos coloca em nenhuma posição privilegiada e permite, inclusiva, a consideração de taxas de expansão maiores do que a velocidade da luz sem contrariar o Princípio da Relatividade Restrita de que a velocidade da luz seja o limite superior de velocidades relativas de sistemas com conteúdo de massa e energia. Pois essa taxa de expansão não se trata de uma velocidade, já que não há, na expansão cósmica, movimento relativo das galáxias. A expansão cósmica permite explicar de modo simples o aumento do comprimento de onda da luz percebida em razão do aumento do espaço em que a luz vai caminhando ao longo do tempo desde que foi emitida.

o universo é um rearranjo do nada?‎

Claro que não! Nunca existiu "o nada". Antes de haver universo não havia nada a ser rearranjado. Nem "o nada". O Universo não proveio do nada. Ele não proveio de nada. Isso significa que não havia nem conteúdo de matéria, campo e radiação, nem tampouco espaço vazio. E nem corria o tempo. Nada não é algo. É a palavra que designa a ausência de qualquer coisa. O Universo não se formou a partir de coisa alguma. Ele surgiu sem ter do que provir. Nem d"o nada".

O senhor envergonha o nosso país com seu ateísmo militante de merda. Com certeza você influencia de uma forma negativa seus alunos. Vou torcer para o senhor levar uma surra dos próprios alunos, velho boiola. Aliás, o que vai fazer quando se encontrar com Jesus ? Vai se converter na hora da morte !!!

Não vou me encontrar com Jesus pois, se ele existiu, já morreu. Quanto a meus alunos, o que faço, quando surge oportunidade, é tirar os antolhos deles para que examinem os fundamentos de suas crenças e das outras, bem como os do ateísmo, para que possam decidir com pleno conhecimento de causa. Isso, absolutamente, não é nada negativo, pelo contrário, é extremamente positivo e saudável. Meus alunos e alunas sempre encararam meu posicionamento a respeito de crenças religiosas de forma inteiramente positiva, pois é uma forma respeitosa para com quem tenha crenças. Jamais afirmei que crentes fossem burros por serem crentes. Apenas que poderiam estar equivocados e que, portanto, precisariam compreender os fundamentos de suas crenças, ou para rejeitá-los, ou para confirmá-los. Tal modo de proceder, absolutamente, não tem nada de vergonhoso.

Anarquismo é impossivel. Não tem ordem nem entre marido e mulher

Bom... talvez por isso é que na concepção anarquista não haja maridos e nem esposas. Todos são de todos. Brincadeiras à parte, o anarquismo é possível sim, mas ao fim de um longo processo educativo em que as pessoas assimilem a ordem espontaneamente.

Sendo ateu, se na hipotética hipótese de se comprovar a existência de deus, aceitaria essa nova crença assim de um momento para o outro? Ou continuaria a ser ateísta? Apesar de nessa hipótese, a existência de um ou mais deuses estar comprovada.‎

Claro que se a existência de algum deus ou de deuses for comprovada eu aceitarei, com toda a tranquilidade. Considero que não existe deus nenhum porque não há evidências e nem provas disso (bem como de que não exista) e os maiores indícios são no sentido da não existência.

Você é contra ou a favor do fim das torcidas organizadas?

Não acho que elas precisem acabar mas sim que precisam se tornar civilizadas. Como estão, não podem ficar. Quem goste de futebol ou outro esporte pode se unir a seus colegas torcedores do mesmo time para incentivar os jogadores e comemorar as vitórias. Mas não podem agredir torcedores de times adversários nem provocar baderna. Esse é mais um ponto que as escolas precisam discutir com os alunos para que eles introjetem as noções de civilidade e boa educação e não se tornem adultos destrambelhados. Pessoalmente não gosto de esportes e uma das razões é a facilidade com que eles despertam instintos selvagens nas pessoas. Acho que exercícios físicos são uma ótima atividade para a saúde, mesmo que não tenha gosto em fazê-los. Mas não gosto que sejam feitos envolvendo competição.

meu amigo é gay este aqui ‎@KingWrestler ele acredita e estuda muito sobre o universo paralelo, ele diz que la ele e homem e é casado tem filhos, mais nesse universo ele é homossexual, diz que deus vai salvar sua alma por que ele no universo paralelo é muito bom e hetero, acho que ele piro, será?

Isso é uma suposição inteiramente sem fundamento da parte dele. Sob três aspectos. Primeiro que Universos Paralelos são uma conjectura não comprovada e provavelmente falsa. Segundo que Deus não existe. Terceiro que se existisse e, por acaso, fosse bondoso, não consideraria que alguém mereceria a danação por ser homossexual, desde que fosse uma pessoa de bem, que amasse ao próximo e não fizesse maldades.

Quando seus pais morreram, você já era ateu? Como foi sua reação?

Sim. Minha reação foi de profunda tristeza, pois eu os amava muito mesmo. Não lamentei por eles, pois, como mortos, não existiam mais e não sentiam mais nada. Mas por mim, minhas irmãs, minha filha e meu filho, meus sobrinhos e todos que gozavam do carinho que eles dedicavam a toda a família, do exemplo que eles davam de pessoas integras e dedicadas a fazer o bem desprendidamente a muita gente desvalida. Dos ensinamentos que eles passavam. Tudo isso não iria haver mais. Meu pai morreu em 1981 e minha mãe em 1988, ele com 64 anos e ela com 69 anos. Novos, portanto. Meu pai morreu de infarto e minha mãe de enfisema. Tudo causado pelo fumo, vício que eles tinham. Por isso vaticino a todo fumante que encontro uma morte em sofrimentos atrozes, para que desistam de fumar.

O que sente uma pessoa quando morre?

Até logo antes de morrer, o que ela sente vai depender do modo como está morrendo. Pode sentir dor ou não. Logo que acaba de morrer não sente mais nada, pois para de funcionar.

A ironia é uma falha de caráter?‎

Não chega a tanto. Mas eu acho uma falta de educação, uma descortesia. Do mesmo modo que o sarcasmo e o deboche. Já o pernosticismo eu acho uma falta de caráter. Como a soberba, a inveja, a ganância, a cobiça, a maledicência, a dissimulação e outros que tais

Quando era criança/adolescente,seus pais o levava na Igreja? Ou você teve uma liberdade para escolher o que queria?

Me levavam à Igreja Católica, na qual fui criado, cursei catecismo, fiz primeira comunhão e tudo o mais. Era um fiel devoto e piedoso. Tanto que eu queria ser santo. Mas tinha liberdade de estudar, criticar e contestar tudo, inclusive religião. Foi, exatamente, o aprofundamento de meus estudos religiosos, filosóficos, científicos e históricos que me levaram a perder a fé e a considerar que fé é algo que não tem cabimento. Estudar sempre foi algo que eu adorava fazer. Meus pais aceitaram minha perda de fé com toda a tranquilidade. Como aceitariam qualquer escolha minha, desde que fosse honesta. Claro que não aceitariam uma escolha pela vida criminosa, por exemplo. Mas aceitariam minha escolha pela homossexualidade, se eu a fizesse. Mas não fiz. Ou pela militância, quer esquerdista, quer direitista. Desde que eu não fosse a favor de nenhuma ditadura, de esquerda ou de direita. E nem de repressão política. Não sei se aceitariam se eu escolhesse pela poligamia, que também não fiz. Mas que considero perfeitamente aceitável, como a homossexualidade, a bissexualidade ou a assexualidade. Meus pais eram intelectuais profundamente liberais. Eu os adorava. Foram meus heróis. Por sua inteligência, cultura, honestidade, correção, caráter e, principalmente, bondade.

Professor quero muito passar em medicina na universidade privada e após ser aprovado no vestibular eu iria solicitar o FIES, acho mais fácil isso do que tentar a federal, estou correto ?

Sim, é mais fácil. Mas não é o melhor. Os cursos das Universidades Públicas, em geral, são melhores e mais exigentes do que os das particulares. Você se tornará um médico mais competente estudando nelas.
Claro que não é a regra geral. Há indivíduos que vão muito além do que é exigido. Mas, em geral, quando se exige pouco, se responde pouco. As particulares costumam facilitar para não dar muita reprovação e não perder alunos. E facilitar é a pior coisa que pode acontecer no estudo, pois diminui o nível de aprendizagem e, logo, de conhecimentos e habilidades dos graduados. Em suma, cria diplomados incompetentes. Isso é péssimo para o povo.

Se alguém nos observasse a cerca de 4.5 bilhões de anos luz de distância, observaria o nosso Sol recém nascido, porque a luz só estaria chegando até lá nesse instante. Mas se o Objeto que eles usassem fosse com uma extrema tecnologia, seria possível observar em tempo real?‎

De modo nenhum. Não interessa a tecnologia de observação que se tenha, só se poderá ver a luz que está chegando lá e ela levou 4,5 bilhões de anos viajando.

Você entende que a venda de TCCs configura crime (ref. quebra de direito autoral, art 184 cp), é apenas um delito "moral" ou sequer é errado? Disserte..

Tanto é um crime, quanto é uma imoralidade e não é ético. Em suma, é um total maucaratismo. Uma pessoa que faça isso é mentirosa, desonesta, vil, abjeta e crápula. Tanto quem compra quanto quem vende. Não merece a menor consideração. Quem compra não pode receber o diploma do curso que está fazendo e quem vende tem que ir para a cadeia. E quem fica sabendo e não denuncia, está compactuando com uma vilania.

"qual a diferença entre um negro e uma lata de bosta? Resp: A lata." Sei que não gosta desse tipo de piada. Qual sua opnião sobre as pessoas que gostam de humor negro mesmo que elas não tenham nenhum tipo de preconceito.‎

Não aprecio humor negro, preconceituoso e nem chulo. Aprecio o humor sutil, inteligente, espirituoso, engraçado, mas que não ofenda nem ridicularize ninguém.

O que você acha do nacionalismo?‎

Não gosto nem do nacionalismo nem do patriotismo. O nacionalismo é mais econômico e o patriotismo é mais político-social. Ambos, contudo, carregam um componente nefasto, que é certo grau de aversão, ou mesmo repúdio, ao que seja estrangeiro. Isso contraria meu pensamento de considerar toda a humanidade como pertencente à mesma comunidade de povos e nações irmanadas. Não digo que seja ruim ter orgulho dos valores nacionais, mas é ruim rejeitar os valores da outras nações. O que precisa ser rejeitado, tanto da própria quanto das outras nações, são os valores equivocados, preconceituosos, mesquinhos e malfazejos de modo geral.

Na sua opinião, qual é a melhor revista sobre atualidades? A Veja é uma boa revista?

Leio a Veja, mas não acho que ela seja boa, porque é tendenciosa. Do mesmo modo que a Carta Capital. Além dessas duas, também leio a Época e a Isto É. Gosto mais da Isto É. Mas leio todas, para me inteirar do ponto de vista de quem está em todos os pontos do espectro ideológico político e econômico, com diferentes visões de mundo, inclusive em outros aspectos, como científico, filosófico, social e psicológico.

Olá, professor. O sr. entende que a vida de animais sencientes possui o mesmo peso da vida de um ser humano?

O mesmo não. Mas têm que ser consideradas como importantes também. A questão só favoreceria a espécie humana no caso de um conflito que determinasse que uma escolha teria que ser feita. Então se escolheria pelo ser humano.

Por que as medições fotoelétricas são muito sensíveis à natureza da superfície fotoelétrica?‎

Porque essa natureza estabelece os níveis de energia de excitação dos átomos do material da superfície, que estabelece que frequências de fótons serão capazes de ionizá-los. Diferentes elementos possuem diferentes potenciais de ionização e, portanto, só são capazes de se ionizarem pela absorção de fótons acima de certo limiar de frequência. Abaixo dele os fótons são refletidos ou transmitidos, mas não absorvidos.

Que mal lhe pergunte, qual é a verdade que o senhor busca?‎

Que bem lhe responda: Quero saber como é que surgiu o Universo, a vida e a inteligência. Não aceito as explicações simplistas das religiões. Elas não têm nenhum fundamento. Então vivo estudando tudo o que se pesquisa a respeito para ir me aproximando da explicação verdadeira, que ainda não se tem, e que eu acho que não será obtida ainda em minha vida e nem em várias décadas de estudos. Mas, quanto mais eu souber a respeito, mais eu fico satisfeito. Da mesma forma, busco saber se há uma razão para que tudo exista ao invés de nada. Pelo que sei, por ora, não há nenhuma. Mas pode ser que haja, só que não acho que seja o capricho de divindade nenhuma, ciosa de louvores. Nisso e na difusão disso, eu encontro um significado para minha vida, já que, em si mesma, a vida só tem por objetivo, simplesmente, viver. Gosto de levar ao mundo a dúvida e o espirito de questionamento, de exame, de reflexão. Para espancar a ignorância e abrir a mente. A respeito de todo e qualquer assunto. Nada está livre de questionamento. Não tenho certeza de nada. Mas tenho minhas convicções, só que elas são abertas a revisão sempre que me convença de estar errado.

Qual é a explicação física para o fenômeno da caixa do leite. Que quando a tampa do leite está para baixo a trajetória do leite fica irregular e que quando está em cima, fica certinha?‎

Porque com a tampa por cima se abre uma passagem para o ar entrar enquanto o leite sai e com a tampa por baixo o próprio leite obstrui a entrada do ar que, assim, acaba penetrando em golfadas, à medida que a saída de leite provoca vácuo no espaço vazio em cima do leite. A entrada das golfadas de ar provoca interrupção do fluxo de saída de leite que, depois, retorna, causando a irregularidade.

O que acontecerá com todos tendo acesso à educação? Quem recolherá o lixo, cavará valas? visto que quando uma pessoa adquire um certo nível de cultura e se forma, ela se torna esnobe e não aceita trabalhos "incompatíveis" com sua formação. E o Sistema não parece criar empregos no mesmo ritmo...‎

Isso não é problema nenhum. Não havendo pessoas simplórias para os trabalhos servis as pessoas instruídas que percam o seu esnobismo e o façam, em sistema de rodízio. Ou então eles ficam sem ser feitos. Não acho que um doutor não possa coletar lixo ou lavar a sua privada, lavar e passar a sua roupa, cozinhar e lavar a sua louça, trabalhar na enxada, trabalhar de pedreiro, ou do que quer que seja. Acho que todo mundo deva ter curso superior mesmo e que todos os serviços, mesmo braçais, sejam feitos por pessoas cultas e instruídas. Ou que fiquem sem ser feitos. Em uma sociedade justa não pode haver pobre nenhum. Todo mundo tem que ser rico, ou, pelo menos da classe média que tenha condições de viver com conforto, numa renda familiar mínima de, pelo menos, uns três mil dólares por mês. Estou falando de renda familiar e não de salário e nem individual.

Mas voce não acha que Getulio Vargas fez coisas boas ? Por exemplo as leis trabalhistas

Sim, acho. Mesmo assim não concordo com ditaduras de modo nenhum. As coisas boas que podem ter sido feitas por algum ditador não justificam nenhuma ditadura. Jamais.

O cálculo 1 foi criado por Newton e aperfeiçoado por muitos , o cálculo 2 foi criado paralelamente ao 1 , com a dúvida dos antigos , ou espécie de continuação do 1 , quais eram os maiores desafios para o 2 ?‎

Se você chama de Cálculo I o cálculo diferencial e integra de funções de uma variável e o Cálculo II o de funções de várias variáveis, o I precede o II, tanto lógica quanto historicamente. Newton foi um dos fundadores do Cálculo. Houve outros, como Leibniz, Barrow, Gauss, Cauchy, Lebesgue, Riemann, Weierstrass, Agnesi (uma mulher) e outros.

Não acha que isso negócio de direita, esquerda é apenas uma coisa só? Acha que poderiam planejar outra forma de governo, ideologia? Uma forma diferente das de hje em dia. Se pudesse ditar um esquema de governo, como seria esse esquema?‎

Não é a mesma coisa não. A direita considera as desigualdades justas e necessárias. A esquerda considera as desigualdades injustas e desnecessárias. A direita, mesmo democrática, cultiva a existência de classes sociais. A esquerda considera que não davam haver. A direita é capitalista, isto é, considera que há quem detenha o capital e quem trabalhe por salário. A esquerda considera que ou todos são assalariados e empregados do governo, que detém todo o capital, no socialismo, ou que ninguém seja assalariado, sendo o capital dividido por todos, no verdadeiro comunismo (que nunca existiu, pois os países que se diziam e dizem comunistas não o são, mas socialistas). Não há relação entre direita e esquerda e democracia e autocracia. Uma e outra tanto podem ser de esquerda quanto de direita. A Social-Democracia é de esquerda. A autocracia da Arábia Saudita é de direita. O melhor esquema de governo é a sua ausência. Mas isso requer um elevadíssimo grau de civilização, que ainda não existe. Até que se chegue lá, sou a favor da Social-Democracia, isto é, uma democracia de esquerda, mista de capitalista e socialista. Como acontece nos países escandinavos.

LinkWithin

Related Posts with Thumbnails